Psych semester review

¡Supera tus tareas y exámenes ahora con Quizwiz!

As you focus in your cell phone conversation while driving, you miss the road sign where you needed to make a turn and drive 20 miles out of the way. This is an example of... A. Inattentional blindness. B. Retinal disparity. C. An apparent-distance illusion. D. Psychokinesis.

A. Inattentional blindness.

According to Chomsky, children around the world use a limited number of patterns in their first year sentences, which strongly suggest that language patterns must be... A. Inborn B. Learned through parental interactions with the child. C. Prelinguistic. D. Related to the intelligence of the kid.

A. Inborn

The action potential is an all or nothing event that... A. Is primarily a electrical process. B. Takes place within the synapse. C. Is primarily a chemical process. D. Takes place at the dendrite receptor sites.

A. Is primarily a electrical process.

If you weigh yourself several times in a row and the bathroom scale gives the same weight each time, then the bathroom scale is said to be... A. Reliable. B. Objective. C. Valid. D. Standardized.

A. Reliable.

The "fight or flight" branch of the autonomic nervous system would be of most interest to a... A. Forensic psychologist. B. Biopsychologist. C. Sensory psychologist. D. Personality theorist.

B. Biopsychologist.

Behavioralist would say that self-defeating behaviors are maintained by immediate reinforcement in the form of... A. Relief from anxiety. B. Ego defense mechanisms. C. Protection from oneself image. D. Avoiding existential anxiety.

A. Relief from anxiety.

Which of the following is NOT considered a basic function of the brain? A. Reordering the genetic code during organogenesis. B. Maintaining bodily functions. C. Directing muscles and glands. D. Creating the magic of consciousness.

A. Reordering the genetic code during organogenesis.

Miguel is a new born. If he is like a majority of infants, the only clearly expressed emotion that he will display is... A. Joy. B. General excitement. C. Sadness. D. Anger.

B. General excitement.

Regarding meaningful material, If one can remember 70 percent of this material after three years, then he or she should remember what percent of this information after 20years... A. 10% B. 30% C. 50% D. 70%

D. 70%

According to the text, __________ refers to the hereditary aspects of one's emotional nature: sensitivity, prevailing mood, and adaptability. A. personality. B. self-concept. C. character. D. temperament.

D. temperament.

True or False... Remembering your social security number would be considered an overt behavior.

False

True or false... In an experiment to find out if taking Vitamin C decreases the number of colds one has during the winter, the number of colds would be the independent variable.

False

True or False... As you are walking down a dark street, your hear a noise directly behind you, and you immediately dart off the sidewalk because of the activation of your amygdala.

True

True or False... The reticular formation does not fully mature until adolescence, which may be why children have such short attention spans.

True

True or False... To be statistically significant, a difference must be large enough so that it would occur by chance in less than 5 experiments out of 100.

True

In the nucleus of every human cell, except for the sperm and ova, are ______ chromosomes. A. 46 B. 60 C. 24 D. 36

A. 46

The life stage known as adolescence is... A. A culturally-defined period between childhood and adulthood. B. A biologically defined event that begins at puberty. C. Recognized as the time between 13 and 18 in all cultures. D. A stage of biological stability but social upheaval.

A. A culturally-defined period between childhood and adulthood.

The single most important thing you might do for a dying person is to... A. Avoid disturbing that person by mentioning death. B. Fill in periods of silence with idle talk during visits. C. Tell that person about the stages of dying. D. Allow that person to talk about death with you.

D. Allow that person to talk about death with you.

Antoine prefers to reason with his children and discuss the limits he sets. His children tend to be very friendly, self-reliant, and socially responsible. According to the research on parental styles, Antoine is exhibiting ___________ parenting. A. Authoritarian. B. Overly permissive. C. Neglectful. D. Authoritative.

D. Authoritative.

Traditional measures of intelligence are closely related to... A. theories of multiple intelligence. B. broad definitions of intelligence. C. neural speed of processing. D. Narrow definitions of intelligence.

D. Narrow definitions of intelligence.

The capacity of our brains to change in response to experience is referred to as... A. Neurogenesis. B. Resiliency. C. Saltatory conduction. D. Neuroplasticity.

D. Neuroplasticity.

Juanita had a stoke and is in the hospital. When her family members visit her in the hospital, Juanita is unable to recognize any of their faces until she hears their voices. Juanita most likely has damage to the underside of her _______ lobes. A. Parietal. B. Temporal. C. Frontal. D. Occipital.

D. Occipital.

Gender identity is... A. defined as the physiological differences between sexual anatomy of males and females. B. indicated by the preponderance of estrogens or androgens. C. determined by the presence of an XX or XY chromosome pair. D. One's subjective sense of being male or female as expressed in appearance, behavior, and attitudes.

D. One's subjective sense of being male or female as expressed in appearance, behavior, and attitudes.

Clarence exerts virtually no control over his son's activities and impulses. According to the research on parental styles, Clarence is exhibiting ________ parenting. A. Authoritarian. B. Authoritative. C. Democratic. D. Overly permissive.

D. Overly permissive.

The primary Somatosensory area is located in the _____ lobes. A. Frontal. B. Temporal. C. Occipital. D. Parietal.

D. Parietal.

In general anxiety related problems involve all of the following EXCEPT... A. pervasive feelings of stress, insecurities, and inferiority. B. a tendency to use rigid defense mechanisms. C. restrictive, self-defeating Behavior patterns. D. alternating episodes of depression and hypomania.

D. alternating episodes of depression and hypomania.

Which of the following receives sensory information very directly and quickly, bypassing the cortex and allowing persons to respond to potential danger before they really know what's happening? A. pons. B. somatosensory cortex. C. corpus callosum. D. amygdala.

D. amygdala.

In a recent study, people were able to detect which types of faces the fastest and why? A. sad faces because we are more sensitive to distress in others. B. neutral faces because ambiguity tends to bother most people. C. happy faces because we are more sensitive to positive emotions. D. angry and scheming faces because we are sensitive to threats of possible harm.

D. angry and scheming faces because we are sensitive to threats of possible harm.

Tanya's parents describe Tanya as the perfect daughter, considerate, obedient, and helpful. Recently, they have noticed that she has lost a considerable amount of weight, is obsessed with dieting and relentless exercise. Although her parents have told her that she has lost enough weight, Tanya still feels like she needs to lose more weight. From these observations, it appears that Tanya is suffering from... A. the behavioral dieting syndrome. B. bulimia nervosa. C. a serious taste aversion. D. anorexia nervosa.

D. anorexia nervosa.

Which of the following is one of the four types of conflicts? A. regression-suppression. B. frustration-aggression. C. displacement-sublimation. D. approach-approach.

D. approach-approach.

Forcible rapists... A. carry out an act that is more brutal than sexual in nature. B. often harbor resentments or outright hatred of women. C. act without concern for the feeling of their victims of guilt over the rape. D. are characterized by all of these.

D. are characterized by all of these.

People with intellectual disabilities... A. respond warmly to love and acceptance. B. can be hurt be rejection, teasing, and ridicule. C. have a right to self-respect and a place in the community. D. are characterized by all of these.

D. are characterized by all of these.

You have been doing lawn work and trimming hedges in the hot sun, and have been perspiring profusely. You will have a(n) _________ thirst. A. intracellular. B. molecular. C. chemically-induced. D. extracellular.

D. extracellular.

Psychologists are attempting to measure which of the following when they study how long it takes people after exposure to a stimulus to react and make a decision? A. metacognition. B. "thin-slicing." C. conditioned responding. D. speed of processing.

D. speed of processing.

Psychiatrist differ from psychologist because psychiatrists... A. Have a masters or PhD degree with special training in psychological theory and research methods. B. Are generally more eclectic than psychologist. C. Extensively trained in the theories and techniques of Sigmund Freud. D. Are Physicians with a specialization in abnormal behavior and Psychotherapy.

D. Are Physicians with a specialization in abnormal behavior and Psychotherapy.

Which of the following categories include risk factors for mental disorders? A. Social conditions such as property. B. Family factors such as abusive parents. C. Psychological factors such as learning disorders. D. All of these.

D. All of these.

The Stanford-Binet Intelligence Scale (SB5) and the Wechsler scales... A. Were both designed to predict a person's chances of success in college. B. Are both individual intelligence tests. C. Are each given to large groups at the same time with minimal supervision. D. Are characterized by all of these statements.

B. Are both individual intelligence tests.

Research findings become more convincing when the results are statistically significant and when the findings... A. Showed a negative correlation. B. Are replicated by other researchers. C. Showed a positive correlation. D. Constitute a natural clinical trial.

B. Are replicated by other researchers.

IBM's Watson supercomputer, which outperforms even expert humans at playing the tv game Jeopardy is an example of... A. Cerebronics. B. Artificial intelligence. C. Proxemics. D. Computerized creativity.

B. Artificial intelligence.

which type of parents help their children grow up with a capacity for live, joy, fulfillment, responsibility, and self-control through positive parent-child interactions? A. Authoritarian parents. B. Authoritative parents. C. Indulgent parents. D. Autocratic parents.

B. Authoritative parents.

Sam has difficulty trusting the women he has dated in recent years. They have all seemed too eager to seek commitment from him. So, when the relationship gets too close, he pulls back and the relationship breaks up. According to the research on adult attachment style, Sam would be described as having a(n) __________ attachment style. A. independent. B. Avoidant. C. Secure. D. Ambivalent.

B. Avoidant.

The Dove Counterbalance Intelligence Test differs from the Stanford-Binet and Wechsler tests in that it is... A. Valid for all ethnic groups, whereas the other tests are not. B. Based on a different operational definition of intelligence. C. More reliable than the Stanford-Binet and Wechsler tests. D. a culture-free test.

B. Based on a different operational definition of intelligence.

When Jared is scolded by his mother for misbehaving, he actually increases the misbehavior because he finally has her attention. For Jared, the scolding served as a... A. Punisher. B. Positive reinforcer. C. Negative reinforcer. D. Response cost.

B. Positive reinforcer.

A good way to encourage development of a child's intellectual capacity is to... A. Require highly abstract thought, regardless of the child's age. B. Present challenges slightly beyond the child's current level of comprehension. C. Present problems that are always highly novel or unusual. D. Expose the child to advanced intellectual topics and styles of thought.

B. Present challenges slightly beyond the child's current level of comprehension.

The time during which you decide if a situation is relevant to you and whether the situation is threatening or a positive challenge is known as... A. the reaction stage. B. Primary appraisal. C. the decision stage. D. secondary appraisal.

B. Primary appraisal.

According to Vygotsky, the range of tasks a child cannot yet master alone but that she or he can accomplish with the guidance of a more capable partner is known as the zone of... A. Task learning. B. Proximal development. C. Cognitive development. D. Vicarious learning.

B. Proximal development.

Which Theory holds that depression is caused by repressed anger turned inward as self-blame? A. Behavioral theory. B. Psychoanalytic theory. C. Existential theory. D. Cognitive theory.

B. Psychoanalytic theory.

Every time you give advice to a friend, she suddenly turns cold and rude toward you. Lately, you've stopped offering her advice because giving your friend advice has been... A. Negatively reinforced. B. Punished. C. Classically conditioned. D. Affected by all of these.

B. Punished.

Jana is an easy baby who smiles and vocalizes frequently, which encourages her parents to smile and talk to her more. This, in turn, causes Jana to smile and teract with her parents more. This illustrates... A. A parental behavior that would occur even if Jana had a difficult temperament. B. Reciprocal influences. C. Genetic predispositions. D. All of these.

B. Reciprocal influences.

Meridian Community college has a total of 4,000 students. One hundred of these students are surveyed about the programs offered at the college. These 100 students surveyed would constitute the... A. Control group. B. Representative sample. C. Independent group. D. Population.

B. Representative sample.

Needle inserted into the body during the acupuncture procedure may relieve pain because the pain... A. Blocks the functioning of pain-relieving neurons. B. Results in the release or neuropeptides. C. Suppresses the production of all neurotransmitters. D. Paralyzes the central nervous system.

B. Results in the release or neuropeptides.

Research has shown that many adults can think formally about some topics, but display concrete thinking when the topic is unfamiliar. This illustrates that formal operation thought... A. Is genetically-programmed. B. Results more from learning and culture. C. Is based on maturation. D. Is more of a right-hemisphere activity.

B. Results more from learning and culture.

The layer of light-sensitive pixels in the digital image sensor of a camera is analogous to the layer of photoreceptors of which area of the eye? A. Pupil. B. Retina. C. Lens. D. Iris.

B. Retina.

Harriet has lost the ability to recognize faces, and she has also lost the ability to detect the emotions that other people are feeling. You would expect to find damage to her... A. Midbrain. B. Right hemisphere. C. Cerebellum. D. Left hemisphere.

B. Right hemisphere.

Psychology is different from the fields of history, law, and business because psychology relies on which of the following to answer questions about behavior? A. Commonsense reasoning. B. Scientific observations. C. Anecdotal evidence. D. Direct philosophic inquiry.

B. Scientific observations.

If you were to observe the development of psychomotor abilities in infants across cultures, you would find the most similarities in the... A. Ages at which each ability appears. B. Sequence in which abilities appear. C. Child-rearing practices used to foster development. D. Relationship between intellectual and psychomotor development.

B. Sequence in which abilities appear.

The CT scans and MRI scans both suggest that the brains of persons suffering from schizophrenia have... A. Become swelled by excess fluid. B. Shrunk (atrophied). C. Become overactive in the frontal lobes. D. Tightly-packed surface fissures.

B. Shrunk (atrophied).

Csikszentmihalyi states that we should "make a commitment to doing things well; seek challenges, follow what sparks out interests, take time for thinking and relaxing, and start doing more of what we really enjoy and less of what we dislike." If we follow these recommendations, he believed we will become... A. self-actualized. B. more creative. C. more able to dig deeper into logic. D. more independent and less controlled by others.

B. more creative.

True or False... After seeing a friend of yours being uncharacteristically quiet, you decide that your friend must be depressed. You later find out that your friend is not depressed but sleepy from staying up all night working on a term paper. You made this error because you failed to distinguish between observation and inference.

True

The functionalists were interested in how thoughts, feelings, perceptions, and habits help people and animals... A. Adapt to their environment. B. Form meaningful Gestalts. C. Introspect and experience the phi phenomenon. D. Form connections between stimuli and responses.

A. Adapt to their environment.

Research has found the healthiest, happiest older people... A. Are connected with others. B. Keep up their physical strength and agility. C. Live in a nursing facility with 24-hour care. D. Hold grudges, which helps maintain memory.

A. Are connected with others.

Paul, an 18 year old college student, is playing online Blackjack end credits his skill for the wins and blames his losses on bad luck. Paul exhibiting which of the cognitive distortions that lead to gambling addictions? A. Attribution error. B. The gambler's fallacy. C. Overinterpretation of cues. D. Selective memory.

A. Attribution error.

The part of the nerve cell that carries information away from the cell body to other neurons is the... A. Axon. B. Synapse. C. Soma. D. Dendrite.

A. Axon.

Schizophrenia may be described as a split... A. Between thought and emotion. B. In personality. C. Between delusion and hallucination. D. In intellect.

A. Between thought and emotion.

Jack and Martha are taking the Scholastic Assessment Test. Based on their sex, we... A. Cannot predict individual performances based on gender. B. Can predict that Martha will be better than Jack at language skills. C. Can predict that Jack will be better than Martha at math skills. D. Can predict that both will perform equally well on both math and language skills.

A. Cannot predict individual performances based on gender.

Both the hereditary as well as environmental influences on the development of schizophrenia in the Genain quadruplets was documented using which research method? A. Case study. B. Correlational study. C. Survey method. D. Experimental method

A. Case study.

Jeffery is 18 months old, and his mother has spent almost every waking moment of the last three weeks trying to toilet-train Jeffery. However, Jeffery has shown no improvement. Jeffery's mother should be told that toilet training as well as other milestones in development are governed by the... A. Child's readiness. B. Quality and quantity of the training. C. Habituation of the child. D. Child's assimilation pattern.

A. Child's readiness.

Compared to NREM, dreams in REM tend to be... A. Clearer and more detailed. B. More rational and reality-based. C. Shorter. D. Characterized by all of these.

A. Clearer and more detailed.

Psychologist who try to directly change thinking patterns that lead to trouble some emotions or behaviors are the _________ therapist. A. Cognitive. B. Behavioral. C. Psychodynamic. D. Humanistic.

A. Cognitive.

Development psychologists are interested in changes in behavior that occur from... A. Conception on to death (the womb to the tomb). B. Birth to adolescence (infant to teen). C. Birth to adulthood (child to grown-up). D. Conception to senescence.

A. Conception on to death (the womb to the tomb).

If an unborn child is exposed to X-rays, PCPs, or drugs taken by the mother, the child could be born a... A. Congenital problem. B. Cephalocaudal pattern problem. C. Proximodistal pattern problem. D. Genetic Defect.

A. Congenital problem.

The man describe he was attempting to control his kleptomania by repeatedly and vividly imagining himself being arrested, handcuffed, taken to jail, and facing his wife's anger and his sons disappointment was using... A. Covert sensitization, a form of behavior therapy. B. Avoidance therapy, a form of REBT. C. Overt sensitization of form of existential therapy. D. Escape therapy, form of insight therapy.

A. Covert sensitization, a form of behavior therapy.

You tell your parents about the " funny stories" that your psychology teacher uses to explain various concepts and theories. your parents respond that they are not sending you to college to hear "funny stories" and that the teacher should lecture on a subject like when they went to college. Which barrier to a problem solving is illustrated by your parents; comment? A. Cultural barrier. B. Perceptual barrier. C. Emotional barrier. D. Learned barrier

A. Cultural barrier.

Divergent thinking is a characteristic of vivid waking fantasies called... A. Daydreams. B. Conventional thoughts. C. Intuition. D. Eidetic imagery.

A. Daydreams.

A minimum change in sensory intensity that is noticeable to an observer and can be coded into neural... A. Difference threshold. B. Transduction limit. C. Absolute threshold. D. psychophysical equation.

A. Difference threshold.

A psychologist who is "eclectic" can best be described as... A. Drawing from many psychological approaches. B. Cognitive rather than behavior. C. Rejecting determinism in favor of free will. D. Preferring pseudo-psychological approaches.

A. Drawing from many psychological approaches.

Zelda is talking to her grandmother on the phone. You know that Grandma has asked Zelda a question because Zelda is nodding. You tell Zelda that Grandma cannot see her nodding, so Zelda just nods her head more vigorously. She later tries to "show" her grandmother her new doll by holding it up in front of the phone. Zelda's inability to see the world except from her own viewpoint is referred to as... A. Egocentric thought. B. Conservation. C. Formal operation thought. D. Object permanence.

A. Egocentric thought.

The professor asks a general question of the class, and although you think you might know the answer, you don't raise your hand to answer because you're afraid that if you're wrong, the other students might laugh at you. You are experiencing which barrier to problem-solving? A. Emotional. B. Learned. C. Perceptual. D. Cultural.

A. Emotional.

Critical thinking is used in psychology to evaluate theories by collecting... A. Empirical evidence. B. Intuitive data. C. Introspective analyses. D. Expert opinions.

A. Empirical evidence.

Well client centered therapy seeks to uncover a " true self" behind the screen of defenses, existential therapy emphasizes... A. Free will and human ability to make choices. B. Unconscious drives and conflicts. C. the need to rebuild thinking, feeling, and acting into connected wholes. D. That one's irrational beliefs tend to cause emotional problems.

A. Free will and human ability to make choices.

Phineas Gage underwent dramatic personality changes after he experienced serious damage to the ______ lobe. A. Frontal. B. Parietal. C. Occipital. D. Temporal.

A. Frontal.

Which theory explains how one type of pain will sometimes cancel another? A. Gate control theory. B. Sensory adaptation theory. C. Sensory conflict theory. D. Lock-and-key theory.

A. Gate control theory.

Concerning gender role stereotypes, which of the following statements is TRUE? A. Gender role stereotypes are simplified beliefs about what men and women are actually like. B. Gender role stereotypes are true reflections of how men and women actually perform on various traits. C. Gender role stereotypes reflect real biological differences. D. Gender role stereotypes are almost always accurate.

A. Gender role stereotypes are simplified beliefs about what men and women are actually like.

Regarding the effectiveness of therapy, which of the following statements is FALSE? A. In a national survey, it was found it only 5 out of 10 people who have sought Mental Health Care say their lives improved as a result of the treatment. B. Someone who feels better after 6 months of therapy may have experienced a spontaneous remission. C. Studies have shown that are more effective for specific disorders. D. When experiments are used to determine Effectiveness for therapies, a waiting-list control group is often used.

A. In a national survey, it was found it only 5 out of 10 people who have sought Mental Health Care say their lives improved as a result of the treatment.

If damage to a particular part of the brain consistently leads to a particular loss of function, then we say that the function is______ in that structure. A. Localized. B. Common. C. Generalized D. Deintegrated.

A. Localized.

For the last several years, everything is looked bleak and hopeless to Felisa. She feels like a failure at work and a failure as a wife and mother. Recently she stopped going to work, and it's become extremely withdrawn from her family, spends most of her time in bed, and has been having frequent suicidal thoughts. Felisa would most likely be diagnosed with... A. Major depressive disorder. B. Schizophrenia. C. Cyclothymic disorder. D. Dysthymic disorder.

A. Major depressive disorder.

Regarding the treatment and prognosis of mental disorders, which of the following statements is/ are TRUE? A. Many milder mental disorders can be successfully treated. B. A major depression or psychotic episode inevitably leads to a lifelong dysfunction. C. Major mental disorders rarely respond well to drugs or any other technique. D. All of these statements are true.

A. Many milder mental disorders can be successfully treated.

Which of the following involves placing a person in a protected setting or medical therapy for emotional disorders is provided? A. Mental hospitalization. B. Deinstitutionalization. C. Logotherapy. D. Pharmacotherapy.

A. Mental hospitalization.

Psychologists who want to study behavior as it unfolds in natural settings use a technique called... A. Naturalistic observation. B. Correlational studies. C. The survey method. D. The clinical method.

A. Naturalistic observation.

Large bundles of axons and dendrites visible to the unaided eye are... A. Nerves. B. Neurons. C. Neurotransmitters. D. Neurilemma.

A. Nerves.

Each night as Leo gets in bed, he has 3 intrusive, unwanted thoughts that he left the stove on or the door unlocked. So he gets up and checks, but once back in bed, he thinks that he might have accidentally turned on the stove or unlock the door while he was choking them and he goes back to the kitchen again. Leah was suffering from a condition known as __________ disorder. A. Obsessive-compulsive. B. Somatization. C. Generalized anxiety. D. Paranoid delusional.

A. Obsessive-compulsive.

To be a "sports car," a vehicle must meet the criteria to be classified as "a car" and must also have a "sporty" look. Therefore, a "sports car" would be considered a _________ concept. A. conjunctive. B. disjunctive. C. relational. D. prototypical.

A. conjunctive.

Reality testing is defined as... A. Obtaining additional information to check the accuracy of one's perceptions. B. Changes in perception that can be attributed to prior experience. C. An initial guess regarding how to organize a stimulus pattern. D. The well-established patterns of perceptual organization and attention.

A. Obtaining additional information to check the accuracy of one's perceptions.

In Thorndike's law of effect, events critical for conditioning... A. Occur after the response. B. Occur before the response. C. Occur simultaneously with the response. D. Are unrelated to the response.

A. Occur after the response.

The graph shown depicts a ______ correlation. A. Perfect positive. B. Perfect negative. C. Zero D. Causal.

A. Perfect positive.

Which type of psychology focuses on our potential for optimal functioning and addresses such topics as love, happiness, creativity, well-being, self-confidence, and achievement... A. Positive psychology. B. Sociocultural psychology. C. Psychodynamic psychology. D. Cognitive psychology.

A. Positive psychology.

Ms. Campbell gives her students a star for every completed homework assignment. The star serves as a... A. Positive reinforcer. B. Negative reinforcer. C. Primary reinforcer. D. response cost.

A. Positive reinforcer.

Traditional psychoanalysis can be objectively criticized as... A. Requiring large amounts of time and money. B. Effective only in treating psychosis. C. Being an effective for a majority of his patients. D. placing too much responsibility on the client.

A. Requiring large amounts of time and money.

Psychology is best defined as the... A. Scientific study of behavior and mental processes. B. Study of individual differences in the group behavior of humans and animals. C. Scientific study of the relationship between mind and body. D. Empirical study of the human personality.

A. Scientific study of behavior and mental processes.

Simon is in his 50s and is completing a test that involves an assessment of vocabulary as well as a timed test that requires Simon to put picture designs together. If we compare Simon's scores on these two tests to these same test taken by Simon when he was 20, we would expect his... A. Score on the vocabulary test to have improved and his score on the picture design to have declined. B. Score on the vocabulary test to have declined and his score on the picture design to have improved. C. His scores on both tests to have declined. D. His scores on both tests to have improved.

A. Score on the vocabulary test to have improved and his score on the picture design to have declined.

Moral values are especially likely to come into sharper focus during adolescence and the transition to adulthood, as capacities for which of the following increase? A. Self-control and abstract thinking. B. initiative and industriousness. C. Intuition and generativity. D. Animism and egocentrism.

A. Self-control and abstract thinking.

The importance of heredity stems from the fact that it... A. Sets the limits of development by providing a framework or potentials and limitations. B. Determines the overall health of the organism. C. Is not subject to environment influences. D. Is more fragile than the environment and, consequently, can be more changeable.

A. Sets the limits of development by providing a framework or potentials and limitations.

Regrading sex as a biological motive, which of the following statements is/are TRUE? A. Sex is necessary for group survival. B. Sex is necessary for individual survival. C. The human sex drive is largely homeostatic. D. All of these statements are true.

A. Sex is necessary for group survival.

In differentiating between the terms sex and gender, A. Sex refers to biological aspects of identity, and gender refers to psychological and social identity. B. Both terms gender and sex refer to one's sociocultural identity. C. sex refers to socioculutral identity, and gender refers to biological identity. D. none of these represent the terms correctly.

A. Sex refers to biological aspects of identity, and gender refers to psychological and social identity.

After a lifetime of blindness, a newly-sighted man walks toward what he thinks are toy cars on the sidewalk. However, as he gets closer the cars seem to grow. This man lacks... A. Size constancy. B. Retinal disparity. C. Brightness constancy. D. Shape constancy.

A. Size constancy.

Helen's mother took three-year-old Helen to a birthday party in the neighborhood. Although her mother encouraged her to play with the other children, Helen did not say a word to any child but sat quietly by her mother watching the other children. Thomas and Chess would probably describe Helen as a(n) ___________ child. A. Slow-to-warm-up. B. Easy. C. Difficult. D. Immature.

A. Slow-to-warm-up.

Which motives are acquired in complex ways, such as cultural conditioning, and include the desire for status, money, success, achievement, and dominance? A. Social motives. B. Primary motives. C. Stimulus motives. D. Homeostatic motives.

A. Social motives.

By eight to 12 months of age, infants smile more frequently when another person is nearby. This behavior is called a(n)... A. Social smile. B. Social intention. C. Moro reflex. D. Automatic smile.

A. Social smile.

Your instructor announces a "pop" test. You have not read the chapter yet. Your heart and breathing rate increases, and your mouth becomes dry. Which nervous system was responsible for this reaction? A. Sympathetic. B. Central. C. Parasympathetic. D. Somatic.

A. Sympathetic.

Which of the following is a small, football-shaped structure in the forebrain that acts as a final switching station for most incoming sensory information on their way to the cortex? A. Thalamus. B. Cerebellum. C. Hippocampus. D. Reticular formation.

A. Thalamus.

And existential therapist would view unhealthy anxiety as... A. The loss of meaning in one's life. B. A raging conflict among the id, ego, and superego. C. The effects of learning and conditioning. D. The result of distorted thinking.

A. The loss of meaning in one's life.

Although almost everyone who has tried to play a poker game like Texas Hold 'Em begins at the mechanical, trial-and-error level with those who persist and can keep up with the other players... A. understanding the general properties of the game. B. utilizing representativeness heuristics in placing bets. C. using algorithms in playing the game. D. using rote methods, such as printed odds tables for each stage of the game.

A. Understanding the general properties of the game.

Research on "bystander apathy" reveals that people often fail to help when other possible helpers are nearby due to a "diffusion of responsibility." Explaining this perplexing problem meets which goal of psychology? A. Understanding. B. Control. C. Description. D. Prediction.

A. Understanding.

I-messages tell children... A. What effect their behavior has on you. B. What's "wrong" with them. C. That you are only concerned with your own needs. D. That you are sensitive to their needs.

A. What effect their behavior has on you.

Bart is described as hard-driving, highly competitive, and achievement-oriented. He tends to race the clock in a self-imposed urgency and get angry when "things do not go as planned." Bart would be described as having... A. a Type A personality. B. a Type B personality. C. a generalized anxiety disorder. D. hypochondriasis.

A. a Type A personality.

Karen Horney believed that emotional health reflects... A. a balance in moving toward, away from, and against others. B. the ability to overcome feelings of inferiority through compensation. C. achieve a balance between one's introversion and extroversion. D. a congruence between one's real self and ideal self.

A. a balance in moving toward, away from, and against others.

The tendency to perceive a problem in a way that binds us to possible solutions is called... A. a mental set. B. a prototype. C. divergent thought. D. brainstorming.

A. a mental set.

Subjects who scored high on both masculine and feminine items of the Bem Sex Role Inventory tended to be more... A. adaptable. B. shy and withdrawn. C. indecisive. D. ambitious.

A. adaptable.

According to David PErkins, reflective intelligence involves... A. an ability to become aware of one's own thinking habits. B. specialized knowledge and skills acquired over time. C. the speed and efficiency of the nervous system. D. an ability to understand others and social interactions.

A. an ability to become aware of one's own thinking habits.

When you are angry, you may feel anything from rage to simple annoyance, which illustrates that... A. anger, like all emotions, can vary in intensity. B. they type of anger one feels depends on his or her personality. C. anger is a mixture of surprise, sadness, and fear. D. anger is a stimulus motive.

A. anger, like all emotions, can vary in intensity.

If you cannot decide whether to order the steak or the Creole shrimp dish because both are so delicious, you are experiencing a(n) _____________ conflict. A. approach-approach. B. double appraisal. C. approach-avoidance. D. avoidance-avoidance.

A. approach-approach.

The position that there is an ideal level of arousal for various activities and that individuals attempt to keep arousal near this ideal level is called the... A. arousal theory. B. biorhythm theory. C. equilibration hypothesis. D. activation hypothesis.

A. arousal theory.

Kinesics refers to the study of... A. body language. B. the autonomic arousal during emotional states. C. Profiling the criminal mind. D. how memories are encoded through movement.

A. body language.

Relaxation or tensions and like or disliking are the two general messages telegraphed by... A. body positioning. B. gestures. C. facial expressions. D. voice inflection.

A. body positioning.

According to positive psychology emotions tend to... A. broaden our focus. B. narrow our focus. C. create dangerous situations. D. make us content with the status quo.

A. broaden our focus.

Exposing yourself to a wide variety of information is a good way to encourage... A. divergent thinking. B. convergent thinking. C. mental sets. D. "thin-slicing."

A. divergent thinking.

Lydia is good at "reading" facial expressions, tone of voice, and other signs of emotion in others and to sense what others are feeling. In terms of emotional intelligence, Lydia is exhibiting... A. empathy. B. Bidirectional communication. C. congruence. D. emotional conceptualization.

A. empathy.

You are on your way to school; and a car pulls out in front of you and its driver then proceeds to drive 20 miles an hour, slowing down at every bump in the road. Your frustration would be classified as... A. external and social. B. internal and social. C. external and nonsocial. D. external and personal.

A. external and social.

If you like studying where there is more socializing and a higher noise level, you are probably high in the trait of ___________, while if you prefer studying in the quiet cubicles in the library, you are probably high in the trait of _________. A. extroversion; introversion. B. introversion; extroversion. C. maina; depression. D. social tendency; antisocial tendency.

A. extroversion; introversion.

The dimensions of sex which refers to X and Y chromosomes is called... A. genetic sex. B. genital sex. C. hormonal sex. D. gonadal sex.

A. genetic sex.

An importing use of DSM-IV-TR includes being a(n)... A. guide to help psychologist to correctly identify mental disorders and select best therapy. B. Statistical Encyclopedia of the incidence of various forms of violent crime committed by patients with mental disorders. C. Accurate guide used by judges and juries in determining the legal definition of insanity. D. guide to current research articles in the field of psychopathology.

A. guide to help psychologist to correctly identify mental disorders and select best therapy.

According to the neural view of intelligence, individuals with higher measured IQ... A. had faster reaction times to the stimuli. B. had slower reaction times to the stimuli as they carefully thought about the stimuli. C. did not show faster or slower reaction times than people with lower measured IQ. D. showed more erratic reaction times than did individuals with lower measured IQ whose reaction times tended to be more stable.

A. had faster reaction times to the stimuli.

Janet is a college student and thinks she is stupid, worthless, and a failure, despite getting good grades, and being an excellent part-time employee. Janet... A. has an inaccurate self-concept and will most likely experience depression. B. is exhibiting organismic valuing due to unresolved conditions of worth. C. is a introverted person who is looking for sympathy from others. D. will experience neurotic anxiety due to a failure to resolve the Electra complex.

A. has an inaccurate self-concept and will most likely experience depression.

When his dad is fixing things around the house, 5yr old Jamies gets out his toy tool kit and acts like he is fixing things. Jamie even tried to walk and talk like his father does because Jamie wants to be like his father. This illustrates the social learning concept of... A. imitation. B. fixation. C. congruence. D. androgyny.

A. imitation.

The actions of the sympathetic branch of the autonomic system... A. improve one's chances of surviving an emergency. B. calm the body after a person has experienced intense arousal. C. constrict the pupils of the eyes. D. increase digestion.

A. improve one's chances of surviving an emergency.

A quick, impulsive thought that does not typically make use of formal logic or clear reasoning describes... A. intuition. B. deductive thinking. C. the thinking process of people who are geniuses. D. inductive thinking.

A. intuition.

Compared to psychologist, a psychiatrist... A. is able to prescribe drugs with all 50 states within the U.S. B. Charges lower fees than a psychologist. C. can do Psychotherapy, well as a psychologist can only counsel. D. realize on psychodynamic technique, while a psychologist is more client centered.

A. is able to prescribe drugs with all 50 states within the U.S.

Social reinforcement has a strong influence on personality development because... A. it is made up of praise, attention, or approval from others. B. it is made up interactions that expand personality development. C. we model everything we do. D. it is made up of rewards for good grades.

A. it is made up of praise, attention, or approval from others.

The denotative meaning of a word is... A. its dictionary definition. B. its stereotypical definition. C. the exact spelling and pronunciation of any word. D. its emotional or personal meaning.

A. its dictionary definition.

In the "love on a swaying bridge" experiment, the findings that the men interviewed on the high swaying suspension bridge called the lovely interviewer more than the men interviewed on the solid wooden bridge suggest that the... A. men on the swaying bridge interpreted their heightened arousal as attraction to the interviewer. B. men on the swaying bridge were younger then the men on the solid bridge. C. interviewer smiled more at the the men on the swaying bridge than the solid bridge. D. men on the solid bridge were more athletic than the men on the solid bridge

A. men on the swaying bridge interpreted their heightened arousal as attraction to the interviewer.

Sexual orientation refers to... A. one's degree of emotional and erotic attraction to members of the same gender, opposite gender, or both genders. B. a person being erotically attracted to both men and women. C. the belief that heterosexuaility is more natural then homesexauality. D. a person being romantically attracted to the same-sex persons.

A. one's degree of emotional and erotic attraction to members of the same gender, opposite gender, or both genders.

Compulsive or destructive deviations in sexual preferences or behavior are called... A. paraphilias. B. sexual dysfunctions. C. sexual scripts. D. bipolar disorders.

A. paraphilias.

A psychiatrist who believes that the behaviors of his patients result from hidden, or unconscious, thoughts, needs, and emotions is using the __________ model. A. psychodynamic. B. medical. C. behavioral. D. humanistic.

A. psychodynamic.

Pablo is seen walking down the street, shouting at imaginary individuals "to turn it down." When the police stopped him and asked him why he was shouting, he tells them that the transmitter that the CIA implanted in his head is "not working properly" and it's too loud. Pablo most likely has a(n) ______ disorder. A. psychotic. B. Neurotic. C. Somatoform. D. Dissociative.

A. psychotic.

Which of the following would have the LEAST effect on feeling of hunger? A. removing a person's stomach. B. weight dropping below the body's "set point." C. removing the hypothalamus. D. the sugar level of the blood being low.

A. removing a person's stomach.

Sarah goes into her final exams feeling like she doesn't know anything, that she is dumb, and that she is going to fail every exam. She will probably have a more intense reaction to stress than other people because... A. she feels she lacks the ability to cope with the demands of finals. B. she is trying to work off frustrated desires or unacceptable impulses in substitute activities that are constructive or accepted by society. C. in general she has more stress in her life. D. she is undergoing school-related mental, physical, and emotional exhaustion.

A. she feels she lacks the ability to cope with the demands of finals.

In general, people who suffered from chronic illnesses, drinking problems. and unstimulating lifestyles typically showed... A. sizeable losses in intelligence as they aged. B. more creative production, especially in writing and artistic endeavors. C. sizeable gains in intelligence as they aged. D. no significant gains or losses as compared to individuals without these problems.

A. sizeable losses in intelligence as they aged.

The cognitive theory of emotion emphasizes... A. the combined effects of physical arousal and emotional labels. B. that emotions are primarily subjective feelings. C. the role of the thalamus in coordinating emotion. D. the role of feedback to the parasympathetic nervous system.

A. the combined effects of physical arousal and emotional labels.

For humanistic theorists, __________ form(s) the structure of personality, while in the behavioral theories, _________ form(s) the structure of the personality. A. the self; habits and expectancies. B. the id, ego, superego; the self. C. traits; the self. D. habits and expectancies; traits.

A. the self; habits and expectancies.

A graphic representation of an individual's scores on several personality characteristics is referred to as a... A. trait profile. B. somatotypic scatter plot. C. rating scale histogram. D. personality analog.

A. trait profile.

Marian is a college freshman who is so self-critical of herself that she feels like she is a failure when she makes a B on an assignment rather than her usual A. She is so terrified of making mistakes that she kept rewriting an essay during class so many times that she did not finish it. Marian is now worried "sick" that that she will make a "bad" grade on the essay. According to your textbook, Marian would BEST be described having... A. unhealthy perfectionism. B. severe test anxiety. C. a low score on neuroticism. D. the "freshman college jitters."

A. unhealthy perfectionism.

In evaluating a new theory, a psychologist asks the questions "Does the theory adequately explain behavior? Does the theory stimulate new research? Does the theory suggests how to treat psychological disorders?" This psychologist is judging this theory based on its... A. usefulness. B. truthfulness. C. reliability. D. concreteness.

A. usefulness.

Compared with children in developing countries, a child who grows up in the U.S. may be better prepared to take both nonverbal tests and traditional IQ tests because our culture is very... A. visual. B. tactile. C. auditory. D. kinesthetic.

A. visual.

Although they do not suffer from hallucinations, emotional excesses, or personality disintegration scene in the other psychoses, people who experience deeply-held false beliefs about experiences they could occur in real life would be diagnosed with... A. Schizophrenia. B. A delusional disorder. C. A paraphilia. D. Bipolar disorder.

B. A delusional disorder.

What do Somnambulism, sleeptalking, and night terrors all have in common? A. All occur in stage 1 (light sleep). B. All occur during NREM sleep. C. All are closely related to dreaming. D. Each is a cause of microsleep.

B. All occur during NREM sleep.

Rene believes that anybody that states that they are voting for "her" political candidate is totally right, honest, and righteous; while anybody that chooses to vote for the "other" candidate is absolutely wrong, unethical, sinful, and "not her friend anymore." Rene is exhibiting... A. Cognitive dissonance. B. All-or-nothing thinking. C. Dichotomous rationalization. D. prototyping.

B. All-or-nothing thinking.

Sensory adaptation refers to a decrease in sensory response that accompanies... A. Threshold shifts. B. An unchanging stimulus. C. Selective attention. D. Perceptual defense.

B. An unchanging stimulus.

Research Sandra Bem describes persons who had high scores in both masculine and feminine traits as... A. Bisexual. B. Androgynous. C. Asexual. D. Schizophrenic.

B. Androgynous.

You have panic attacks and feel like you are going to die; or you are afraid to do things that most people can do; or you spend unusual amount of time doing things like washing your hands or counting your heartbeats. If you showed these symptoms you would most likely be diagnosed with some form of ________ disorder. A. Somatoform. B. Anxiety. C. Psychotic. D. Dissociative.

B. Anxiety.

Calvin has been on a "pleasure binge" for the last 3 days. He has cleaned out his bank account spending the money on anything that gains his attention. He would be described as loud, elated, and hyperactive. A week ago he was feeling like a failure and was withdrawn and unhappy. Calvin would most likely be diagnosed with... A. Schizophrenia. B. Bipolar disorder. C. Dysthymic disorder. D. Dissociative disorder.

B. Bipolar disorder.

On the day of the test, Eddie was very sleepy, while Petra was in a state of panic about the test. According to the arousal theory, A. both Eddie and Petra will probably perform well since they are both functioning at their optimal arousal levels. B. Both Eddie and Petra will probably perform poorly on the test, because his arousal is too low and hers is too high. C. Both Eddie and Petra will probably perform well on the test, since test-taking is not influenced by arousal levels. D. Petra will probably perform well since she is focused, while Eddie will probably perform poorly due to a low arousal level.

B. Both Eddie and Petra will probably perform poorly on the test, because his arousal is too low and hers is too high.

Hypnosis is characterized by all of the following EXCEPT... A. An increased openness to suggestion. B. Brain waves that resemble sleep. C. relaxation. D. Narrowed Attention.

B. Brain waves that resemble sleep.

Olfaction and gustation are considered... A. Unrelated to each other. B. Chemical senses. C. Of little importance for humans. D. Vestibular senses.

B. Chemical senses.

Which therapy focuses on being authentic, showing empathy, and reflecting the client's thoughts and feelings? A. Psychoanalysis. B. Client-centered therapy. C. Cognitive therapy. D. Gestalt therapy.

B. Client-centered therapy.

Although Mark Look in the dictionary for the definition of the word "liberal," he still would be offensive if someone referred to him as "liberal." Thus, Mark's negative view of the word "liberal" represents, for him, this concept's _________ meaning. A. Disjunctive. B. Connotative. C. Denotative. D. Conjunctive.

B. Connotative.

You are asked, "Erica is shorter than Zoey but taller than Carlo, and Carlo is taller than Jared. Who is the second tallest?" To solve this problem, you must use _________ thinking. A. Intuitive. B. Convergent. C. Creative. D. Divergent.

B. Convergent.

During your professor's lecture on the history of mental illness, you learned that in the past there were disorders called drapetomania, nymphomania, and anarchia, which do not exist today. From this lecture, you have learned that what is often considered normal or abnormal depends on... A. Statistical deviation. B. Cultural disapproval. C. The level of functioning. D. Subjective discomfort.

B. Cultural disapproval.

Since values, language patterns, and traditions greatly affect one's performance on tests, psychologists have tried to develop tests that do not disadvantage certain groups with these tests being called _________ test. A. Diversity B. Culture-fair C. Eco-balanced D. Multicultural

B. Culture-fair

The goals of psychology are to... A. Compare, analyze, and control human behavior. B. Describe, understand, predict and control behavior. C. Research, infer, summarize, and publish. D. Develop effective methods of psychotherapy and cure mental illness.

B. Describe, understand, predict and control behavior.

Deliberately making an environment more stimulating, nutritional, comforting, and loving is referred to as... A. Psychological intervention. B. Enrichment. C. Permissive love. D. Overindulgence.

B. Enrichment

Which type of memory would be used to answer this multiple choice question? A. Implicit. B. Explicit. C. Episodic. D. Contextual.

B. Explicit.

Janie states that "she likes to write, so she will become a journalist," while Matt wants to make a lot of money and decides that "being a lawyer pays well." Both Janie and Matt are making their career decisions by... A. Framing the questions of a career too broadly. B. Framing the questions of a career too narrowly. C. Using divergent, creative approach in selecting a career. D. Using a random search strategy in selecting a career.

B. Framing the questions of a career too narrowly.

Humanistic theories emphasize... A. The use of rewards and punishments to shape behavior. B. Free will and self-determination. C. Internal motives, conflicts, and unconscious forces. D. That stimulus and response connections determine behavior.

B. Free will and self-determination.

Which of the following type of therapist helps people rebuild thinking, feeling, and acting in Connected holes by expanding the personal awareness and by helping them fill in the gaps in experiences? A. Psychodynamic therapist. B. Gestalt therapist. C. Cognitive therapist. D. Client-centered therapist.

B. Gestalt therapist.

Regarding dexterity, there is a greater portion of motor cortex devoted to the... A. Arms. B. Hands and fingers. C. Legs. D. Feet and toes.

B. Hands and fingers.

You are studying for a string of five finals in a row, and you haven't slept in the last two days. You probably... A. Have trouble doing interesting and complex mental tasks. B. Have trouble paying attention and have drooping eyelids. C. Have an increased pain tolerance. D. Exhibit all of these characteristics.

B. Have trouble paying attention and have drooping eyelids.

When an individual develops from a zygote to an embryo to a fetus to a neonate and so on until one's death, this overall pattern of physical development is known as the... A. Adaptation sequence. B. Human growth sequence. C. Senescence pattern. D. Zone of proximal development.

B. Human growth sequence.

Which of the following examples illustrates a natural consequence? A. If Petra is not clean her room, she will not get to go to the movies with her friends. B. If Aaron refuses to do his own laundry, he will have nothing clean to wear. C. If Matthew does not eat his dinner, he will not get dessert. D. If Rhonda does not finish her homework, she will not get to play her video game.

B. If Aaron refuses to do his own laundry, he will have nothing clean to wear.

According to the text, what conclusion can be drawn in the nature-nurture debate? A. Environment is the most important because influences, such as learning, nutrition, disease, and culture affect out ability to develop to out potential. B. If it the interaction between heredity and environment that is important in determining who we become. C. Heredity is the most important because we inherit our potential for development as well as our limitations. D. Neither heredity nor environment is important; only our own behavior determines who we are.

B. If it the interaction between heredity and environment that is important in determining who we become.

Margo takes a test that measures the level of several emotions. Using statistical abnormality as the definition of cycle pathology, Margo would be considered normal or average it's all her scores fell where on the normal curve? A. At the high end of the curve. B. In the middle of the curve. C. At the lower end of the Curve. D. Only if her scores fell outside the Curve.

B. In the middle of the curve.

The advice to " sleep on it" would relate to which stage of creative thinking? A. Illumination. B. Incubation. C. Verification. D. Preparation.

B. Incubation.

Which theory of hearing explains that hearing high or low tones depends upon which area of the cochlea is most strongly stimulated? A. tone B. Place. C. Frequency. D. Amplitude.

B. Place.

After riding in the back seat of a car, making rapid turns along a winding road, you expect dizziness and nausea because... A. Poisons build up in the system which cause vomiting to eject them. B. Information from the semicircular canals and the rest of you body is inconsistent. C. Auditory and visual stimuli combine too rapidly and produce a sensory overload. D. Kinesthetic receptors have been stimulated too vigorously and eventually shutdown.

B. Information from the semicircular canals and the rest of you body is inconsistent.

College students in their 20s who are developing life-long friendships are in which psychosocial stage? A. Industry versus inferiority. B. Intimacy versus isolation. C. Integrity versus despair. D. Generativity versus stagnation.

B. Intimacy versus isolation.

A substance capable of altering attention, judgement, perception, and emotions... A. Is classified as a narcotic. B. Is classified as a psychoactive drug. C. is an addictive drug. D. Produces withdrawal symptoms.

B. Is classified as a psychoactive drug.

The first step in self-directed desensitization is... A. Understanding what cause your anxiety. B. Learning to relax voluntarily. C. Deciding on your most feared stimulus in your hierarchy. D. Deciding on the least feared stimulus in your hierarchy.

B. Learning to relax voluntarily.

A teacher shows Megan two short glasses filled to the same level. While Megan is watching, the teacher pours the water from one of the short glasses into a tall thin glass. When Megan is asked which glass has the most water, the short one or the tall one, Megan replies that "There is the same amount of water in each. The tall glass is just thinner." Megan is... A. Exhibiting object permanence. B. Making a transformation. C. Demonstrating scaffolding. D. Demonstrating consistency.

B. Making a transformation.

On intellectual tests, top scorers over the age 65... A. Scored significantly lower than men younger than 35. B. Matched the average for men younger than 35. C. Scored significantly higher than men younger than 35. D. Made the same score but took twice as long to complete the test.

B. Matched the average for men younger than 35.

Information in long-term memory is generally stored on the basis of... A. Visual images. B. Meaning and Importance. C. How the words sound. D. Patterns of visual and auditory images.

B. Meaning and Importance.

A statistical technique that can be used to combine the results of many Studies, as if they were all part of one large study is referred to as... A. The natural clinical test. B. Meta-analysis. C. Double-blind experimental analysis. D. The standard error of measurement.

B. Meta-analysis.

The power of the brain arises from the cooperation of large numbers of neurons connected together into... A. Neural nodes. B. Neural networks. C. Neurilemmas. D. Synaptic vesicles.

B. Neural networks.

Neurons " talk" to each other chemically when which of the following are released? A. Somas. B. Neurotransmitters. C. Neurilemmas. D. Ion Channels.

B. Neurotransmitters.

Mattie, and extremely overweight patient at a mental hospital, had a persistent and disturbing habit of stealing food Patience. The behavior Therapist assigned Mattie to a special table in the dining room. If Mattie approached any other table, she was immediately removed from the dining room. To produce extension of Mattie's food stealing, that therapist used which of the following regarding this Behavior? A. Aversive conditioning. B. Nonreinforcement. C. Covert desensitization. D. Respondent conditioning.

B. Nonreinforcement.

Regarding death, which of the following statements is FALSE? A. Many people have little direct experience with death until they are fairly old. B. Older people has more deaths fears than younger people. C. Older people more often fear the circumstances of dying, such as pain or helplessness, rather than death itself. D. What appears to be an absence of fear of death main reality reflect a denial of death.

B. Older people has more deaths fears than younger people.

Because Robin has more experience with people from her own race, she is better able to recognize people from her own race because she is more familiar with the features that help us recognize different persons. This illustrates the importance of... A. Sensory adaptation. B. Perceptual learning. C. Habituation. D. Dishabituation.

B. Perceptual learning.

Jasmine is experimenting jet lag after a long flight, that is, her body had difficulty adjusting to the time difference so that she felt sleepy when she should be awake and vice versa. Which gland and hormone are responsible for Jasmine's condition? A. Adrenal gland----norepinephrine. B. Pineal gland----melatonin. C. Thyroid gland----thyroxin. D. Adrenal gland----epinephrine.

B. Pineal gland----melatonin.

As part of a class assignment to "find out how it feels to be crazy for a day," students perform some "strange" Behaviors in public, such as talking to imaginary people, covering their heads with aluminum foil, or walking around in a mall with an open umbrella. Their students would most likely be viewed as abnormal based on... A. Cultural relativity. B. Social nonconformity. C. Depersonalization. D. Neurotic Behavior.

B. Social nonconformity.

Grant is sitting in his high chair calling to his dog, "Here Dog." He than turns to his mother and says, "Want Juice." Grant is Exhibiting... A. the single-word stage. B. Telegraphic speech. C. Cooing. D. Babbling

B. Telegraphic speech.

Amy lifts her head, then learns to crawl, then to walk with the development progressing from her head to her feet. This progression illustrates... A. The proximodistal pattern. B. The cephalocaudal pattern. C. Assimilation. D. Habituation.

B. The cephalocaudal pattern.

Which of the following statements about personality is TRUE? A. After age 40, events such as moving to a new city or finding new friends often reshape one's basic personality. B. The person you are at 30 is, for the most part the person you will be at age 60. C. After age 40, the older a person becomes the more disagreeable and cranky they become. D. By age 12, personality has typically become quite stable.

B. The person you are at 30 is, for the most part the person you will be at age 60.

The moon appears smaller when it is overload than when it is near the horizon because... A. The moon's appearance is magnified by the atmosphere. B. There is a lack of depth cues when moon is overhead. C. Retinal disparity creates the stereoscopic pictorial cues. D. There are too many depth cues present.

B. There is a lack of depth cues when moon is overhead.

Regarding the child-rearing practices of different cultures, which of the following statements is FALSE? A. Child-rearing practices in different cultures reflect different customs and belief systems. B. There is general agreement across cultures about which behaviors are considered "good" and "bad". C. Many AMerican child-rearing customs would be considered strange or inappropriate in other parts of the world. D. Parenting can only be judged if we know what culture or ethnic community a child is preparing to enter.

B. There is general agreement across cultures about which behaviors are considered "good" and "bad".

At what age do children become increasingly capable of mischief and temper tantrums as well as showing their independence by refusing to obey their parents' demands even though they understand what their parents want them to do? A. Nine months of age. B. Two years of age. C. 12 months of age. D. Four years of age.

B. Two years of age.

Which of the following would be a behavior that would help if you were providing support and comfort to a friend in distress? A. Helping the friend to place blame on the person to upset him or her. B. Using open-ended questions and supportive statements. C. Probing painful topics to produce catharsis. D. Showing impatience to encourage your friend to stop feeling "sorry" for him or herself.

B. Using open-ended questions and supportive statements.

Daily annoyances that are closely linked to immediate health and psychological well-being are referred to as... A. conflicts. B. microstressors. C. macrostressors. D. burnouts.

B. microstressors.

Abby is a college student, who enjoys taking challenging classes so that she can learn skills that will be helpful to her in the future. Although she has high A's in all of her courses, she strives to do the best she can on every test and assignment because it is important to her to meet her own standards of excellence. Abby has a high need for... A. power. B. achievement. C. homeostasis. D. all of these.

B. achievement.

Garrett has been feeling fatigued at work and has been experiencing more backaches and necklaces. Garrett believes that he must actively change aspects of his environment that contribute to his stress, such as utilizing stress-management techniques, getting more sleep, exercising more, and improving his diet. Garrett's view of health fits the... A. hypochondriac perspective. B. biopsychosocial model. C. medical model. D. mind-body perspective.

B. biopsychosocial model.

The word shot means different things when we are thinking of marksmanship, bartending, medicine, photography, or golf, which illustrates that words get much of their meaning from... A. grammar. B. context. C. prototypes. D. phonology.

B. context.

The ability to perceive, use, understand, and manage emotions is called... A. kinesic sensitivity. B. emotional intelligence. C. creative thinking. D. intuition.

B. emotional intelligence.

Psychiatrist Daniel Langleben theorizes that a liar must inhibit telling the truth in order to lie; therefore, an fMRI brain image of the person's brain would show... A. a decrease in the activity of the limbic system. B. extra brain areas that have been activated. C. a decrease of brain activity in the frontal lobes. D. More right hemisphere activity than left hemisphere activity.

B. extra brain areas that have been activated.

Too many things to do, not enough money for housing, fear of losing valuables, and parents' expectations are faced by almost every college student and are common examples of... A. conflicts. B. hassles. C. burnouts. D. major life changes.

B. hassles.

When your body falls below its ideal level for body temperature, it will automatically begin to shiver in order to raise your temperature and bring your body back into... A. Thermostatic potential. B. homeostasis. C. incentive levels. D. optimum potential level.

B. homeostasis.

psychologists who believe that people grow and develop throughout their lives and that people are inherently good are... A. social learning theorists. B. humanists. C. radical behaviorists. D. psychoanalysts.

B. humanists.

Which area of the brain contains both the feeding center and the satiety center? A. hippocampus. B. hypothalamus. C. amygdala. D. medulla.

B. hypothalamus.

Meg rarely wears her seat belt and often texts on her phone while she drives. However, she does not believe she will have an accident. Meg is exhibiting which intuitive thinking error? A. the representativeness heuristic. B. ignoring the base rate. C. allowing too much time for incubation of the problem. D. framing a problem too broadly.

B. ignoring the base rate.

Rogers described mismatches between one's self-image and reality as a state of... A. moral anxiety. B. incongruence. C. negative symbolism. D. basic anxiety.

B. incongruence.

Motivation is defined as... A. the goals which reward our behavior and maintain behavior. B. internal mechanisms which initiate, maintain, direct, and terminate behavior. C. an internal deficiency that energizes behavior. D. needs associated with impulses for self-actualization.

B. internal mechanisms which initiate, maintain, direct, and terminate behavior.

For both genetic females and male, hormonal problems before birth may produce an individual who has amigous sextaul anatomy, a condition referred to as a(n) _______ person. A. bisexual. B. intersexual. C. transsexual. D. homosexual.

B. intersexual.

According to Ellis, Which of the following develop(s ) from the following three core ideas: you must treat me fairly; I must perform well and be improved by significant others; and conditions must be the way I want them to be? A. avoidance learning. B. irrational beliefs. C. cases of transference. D. Cases of meaninglessness.

B. irrational beliefs.

Jasmine is attending a program at school that includes practice in stress reduction, self-protection, decision-making, self control, and social skills that will help her solve problems rather than turning to drug use or other destructive behaviors. This program is called... A. a community health campaign. B. life skills training. C. a psychological health workshop. D. refusal skills training.

B. life skills training.

Behavioral theories of personality are often criticized for their... A. negative view of human nature. B. limited recognition of temperament. C. inability to predict behavior, being only able to explain behavior after the fact. D. inability to test or verify concepts.

B. limited recognition of temperament.

Having a high score on the Social Readjustment Rating Scale (SRRS) means that... A. the last stage of exhaustion has been reached. B. one runs a high risk of accident or illness. C. one has a great deal of satisfaction with life. D. ambivalence and vacillation can be anticipated.

B. one runs a high risk of accident or illness.

Strong emotions, such as fear or pleasure, tend to be followed by an opposite emotional state when the stimulus ends with the strength of both emotional states changing over time, according to the ________ theory. A. Yerkes-Dodson. B. opponent-process. C. drive reduction. D. Cannon-Bard.

B. opponent-process.

Which of the following pairs is correct? A. sex-homeostatic. B. pain-episodic. C. food-secondary motive. D. thirst- stimulus motive.

B. pain-episodic.

During the finals weeks of the College semester, Mee Jung has barely managed to survive the rush of make-or-break term papers, project, and classroom presentations and then endorse a week and a half of stress, pressure, and frustration due to final exams. She is now off for the summer and should expect... A. a period of no stress but excessive boredom to occur. B. periods of intense stress to be frequently followed by illness. C. more periods of intense stress during the summer because of her Type A personality. D. none of the these occurrences, since periods of stress and of illness are unpredictable.

B. periods of intense stress to be frequently followed by illness.

Regarding personality disorders, which of the following statements is FALSE? A. Personality disorders are all maladaptive but vary in severity of impairment. B. personality disorders typically emerge abruptly after a traumatic experience. C. the patterns of personality disorders often begin in adolescence or childhood. D. Because personality patterns usually become stable by a young age, this makes personality disorders difficult to treat.

B. personality disorders typically emerge abruptly after a traumatic experience.

In order not to feel poor, Jermaine has committed himself to changing his circumstances through education and hard work. Jermaine is utilizing... A. emotion-focused coping. B. problem-focused coping. C. primary appraisal. D. reaction formation.

B. problem-focused coping.

Each evening before going to bed, Rachel tightens all the muscles in her feet, then voluntarily relaxes them. Then she tightens all the muscles in her legs, then voluntarily releases them. She does this all the way to her head. Rachel is practicing the technique of... A. guided imagery. B. progressive relaxation. C. stress inoculation. D. meditation.

B. progressive relaxation.

Jay's mother is always fussing at him about his messy room, his organized study habits, and his tendency to wait until the last minute to complete an assignment. interestingly, when you observe Jay's mother, she also has these shortcomings. By focusing on her son's bad habits, Jay's mother does not have to focus on her own bad habits. She is using the defense mechanism known as... A. sublimation. B. projection. C. reaction formation. D. compensation.

B. projection.

John's directive and confrontational therapist challenges his irrational belief that he will be a failure in college because he's the first in his family to attend college. His therapist is most likely a(n) ________ therapist. A. Person-centered therapist. B. rational-emotive Behavior therapist. C. psychoanalytic therapist. D. Existential therapist.

B. rational-emotive Behavior therapist.

You are trying to determine which of two terms is related to identical twins. You see "monozygotic" and "dizygotic." You remember that a zygote is formed from one egg and one sperm and that "mono" means one and "di" means two. Identical twins would have to result from the same egg and the same sperm, thus one zygote. Using old information to derive the meaning of "monozygotic" and "dizygotic" illustrates the use of... A. selective encoding. B. selective comparison. C. selective attention. D. selective combination.

B. selective comparison.

The textbook problem of how many socks to take out of a drawer to get a matched pair when the drawer contains a four to five mixture of white and black socks requires the reader to realize that the four to five mixture is irrelevant information and that the only relevant information is that there are two colors of socks. Getting the correct answer of three socks requires the reader to first use which of Davidson's insight abilities? A. selective comparison. B. selective encoding. C. selective attention. D. selective combination.

B. selective encoding.

Cindy is on a first date with Jacob. Cindy considers this first date to be a "friendly, get-to-know-you" type of experience, whereas Jacob is expecting this first date to quickly move to a sexual encounter. When the date ends at Cindy's apartment door, both Cindy and Jacob are angry, hurt, and vow never to see each other again. This incompatibility is due to their having different... A. expressive behaviors. B. sexual scripts. C. sensate focus. D. sexual orientations.

B. sexual scripts.

According to psychologists who support the feedback hypothesis of emotion, if you want to be happy, you should... A. shape you own moods through self-reinforcement. B. smile. C. seek friends with similar values. D. surround yourself with others who are happy.

B. smile.

The fact that "dog bites man" has a very different meaning from "man bites dog" demonstrates the importance of... A. connotation. B. syntax. C. linguistic determinism. D. conditional relationships.

B. syntax.

After night class, you are walking back to you car when you catch a glimpse of someone running up behind you. You quickly turn and brace for a possible attack, but the students runs on by. It is only after you reacted to the possible attack that you realized that you felt afraid. Reacting before experiencing the feeling of fear is BEST explained by which theory of emotion? A. Schachter's cognitive theory. B. the James-Lange theory. C. the common sense theory. D. the Cannon-Bard theory.

B. the James-Lange theory.

Marian would be described as physically and psychologically healthy. She is happy, optimistic, and self-confident, and is able to bounce back emotionally from adversity. According to health psychologists, Marian is in a state of... A. self-actualization. B. wellness. C. stress inoculation. D. incongruence.

B. wellness.

Morgan finds riding roller coaster thrilling and fun, while her friend Abby finds this activity distressing and terrifying. Thus, to know if an activity causes a person eustress or distress depend on... A. whether the person has a Type A or Type B personality. B. what meaning the person places on events. C. whether the somatic or autonomic nervous system is involved. D. how each person's limbic system interprets the situation.

B. what meaning the person places on events.

Regarding patterns of suicide, which of the following statements is TRUE? A. suicide is the number one cause of death among 15-24 year olds. B. white male 65 years and older have a high risk of committing suicide. C. Single person's have lower rates of suicide than married persons do. D. In the United States, African Americans have a higher rate of suicide then do caucasians.

B. white male 65 years and older have a high risk of committing suicide.

Regarding the functioning of the left and right hemispheres of the brain, which of the following statements is FALSE? A. Some task may make more use of one Hemisphere or the other. B. People normally use both sides of the brain at all times. C. A smart brain is one that sees the details first and then works it into an overall pattern. D. Each hemisphere does the parts of the activity it does best and then shares the information with the other side.

C. A smart brain is one that sees the details first and then works it into an overall pattern.

Regarding the midlife crisis, which of the following statements is TRUE? A. Research shows that no one really experiences a "midlife crisis." B. Research show that almost everyone has significant difficulty during the midpoint of life. C. About a quarter of men and women believe they have experienced a midlife crisis. D. Approximately one-half of men and women believe they have experienced a midlife crisis.

C. About a quarter of men and women believe they have experienced a midlife crisis.

A nursing home administrator gives the residents little choice in activities the participate in or in the food that is served. Friendships between opposite sex residents are also discouraged. She tells the staff that "the elderly are like little children that don't know what's best for them." This statement of the administrator reflects... A. The old age syndrome. B. The current research findings on successful aging. C. Ageism. D. The type of scaffolding utilized with the elderly.

C. Ageism.

An important outcome of Sperry's split-brain research was... A. A technique for training enhanced creativity. B. A procedure for treating schizophrenia. C. An understanding of hemispheric specialization. D. Insight into the relationships between the central and peripheral nervous systems.

C. An understanding of hemispheric specialization.

People suffering from which of the following seem to generate their own misery, regardless of what is happening around them and feeling that they must be on guard against future threats that could happen at any time? A. Nervous breakdowns. B. Adjustment disorders. C. Anxiety disorders. D. All of these.

C. Anxiety disorders.

A teratogen is... A. The cause of cystic fibrosis and sickle-cell anemia. B. Another name for a genetic defect. C. Any substance capable of causing birth defects. D. Characterized by all of these.

C. Any substance capable of causing birth defects.

A smile that involves not only the mouth, but also the small muscles around the eyes would be a(n) ____________ smile... A. Social. B. Forced. C. Authentic. D. Intentional.

C. Authentic.

Which of the following statements is FALSE? A. Nonconformity does not always indicate a disorder, and Conformity is not a sure sign of mental health. B. Within a culture, a behavior may be defined as abnormal in one situational context but not another. C. Behaviors defined as abnormal and one culture are, by definition, abnormal in all cultures. D. In all cultures consistent failure to communicate or behave predictably is considered abnormal.

C. Behaviors defined as abnormal and one culture are, by definition, abnormal in all cultures.

In what ways are trephining and exorcism alike? A. both cause mental illness. B. Both were used as treatments by Pinel in the Bicetre asylum. C. Both were considered approaches for removing evil spirits. D. both are considered Gestalt Therapies.

C. Both were considered approaches for removing evil spirits.

Sensitive periods in development... A. Usually involve cries and conflict. B. Dramatic but temporary changes in behavior. C. Can permanently alter the course of development. D. Are almost always characterized by emotional upheaval.

C. Can permanently alter the course of development.

Messages flow from the brain to the spinal cord and then to other parts of the body through the _____ nervous system. A. Peripheral. B. Temporal. C. Central. D. Parietal.

C. Central.

Which of the following statements concerning the sexual revolution is TRUE? A. The U.S. is undergoing a wholesale return to less liberal times regarding sexual attitudes and sexual behaviors. B. Sexual behavior has increased in the last 40 years to very high levels of sexual promiscuity. C. Changes in attitudes are still larger than changes in actual sexual behavior. D. A conservative countermovement has basically halted sexual liberalization.

C. Changes in attitudes are still larger than changes in actual sexual behavior.

Like most people with social phobia, Bonnie is a perfectionist and is excessively concerned with making mistakes, received criticism where there is none, avoid social situations that focus too much attention on her, and tends to have distorted that that she has failed even when she is successful. This explanation of Bonnie's social phobia is consistent with which view of anxiety disorders? A. Psychodynamic. B. Behavioral. C. Cognitive. D. Humanistic-existential.

C. Cognitive.

According to Haier's PET scan research, persons who do well on difficult reasoning tests have brains that... A. Are the easiest to study using EEGs. B. Have the largest corpus callosums. C. Consume the least glucose. D. Are the most active and consume the most glucose.

C. Consume the least glucose.

Nerves that leave the brain directly are called ______ nerves. A. Sympathetic. B. Peripheral. C. Cranial. D. Spinal

C. Cranial.

A person's developmental level refers to that person's... A. Past state of health and physical development. B. Future capabilities and potentials regarding intellectual development. C. Current state of physical, emotional, and intellectual development. D. Future status in all developmental areas based on past measurements of health and well-being.

C. Current state of physical, emotional, and intellectual development.

Which of the following persons would most likely be diagnosed with an antisocial personality disorder? A. Adam is perfectionist who follows over routine and prefers to be in control at all times. B. Tommy is deeply mistrustful and suspicious of other's motives. C. Darden is charming, but irresponsible and lacked guilt or remorse. D. Walter is timid and uncomfortable in social situations, and fears evaluation.

C. Darden is charming, but irresponsible and lacked guilt or remorse.

Rats raised in environments that had cages decorated with colorful patterns and filled with platforms, ladders, and cubbyholes... A. Became more aggressive and territorial in their behavior. B. Became confused and withdrawn. C. Developed a larger, heavier brain with a thicker cortex. D. Were no different in physical or behavioral characteristics than the rats raised in the standard laboratory cages.

C. Developed a larger, heavier brain with a thicker cortex.

Statistical approaches to abnormality define as "abnormal" those who... A. show evidence of loss of contact with reality. B. Are on happy, with John, and depressed. C. Deviate from typical or average patterns of behavior. D. Are disabled with anxiety.

C. Deviate from typical or average patterns of behavior.

In baseball an "out" can occur when a player strikes out, if the player is tagged out while running the bases, or if he hits a ball that is caught by an outfielder. Thus, the concept of an "out" would be considered a __________________... A. Conjunctive. B. Prototypical. C. Disjunctive. D. Connotative.

C. Disjunctive.

Dr. Guerillmo Uses whatever message best fit a client's particular problem and often seeks to combine the best elements of various Therapies in all aspects of his therapy. Dr. Guerillmo Would be considered a(n) ________ therapist. A. Existential. B. Convergent. C. Eclectic. D. Divergent.

C. Eclectic.

When a researcher creates two groups, varies a condition, and records whether varying the condition had any effect on behavior, the researcher is conducting a(n)... A. Correlational study. B. Naturalistic observation. C. Experiment. D. Comparative case study.

C. Experiment.

When experiments are used to determine whether therapy is effective, the therapy can be deemed effective only if the... A. control group shows Improvement, but the experimental group shows none. B. Waiting list group shows dramatic improvement over the experimental and control groups. C. Experimental group shows more Improvement than the control group. D. Experimental and waiting list groups show Improvement and the control group shows no improvement.

C. Experimental group shows more Improvement than the control group.

Mozart produced more than 600 pieces of music. Shakespeare wrote 154 sonnets, and Salvador Dali created more than 1500 works of art. This outpouring of ideas represents these creative geniuses'... A. novelty. B. Originality. C. Fluency. D. Flexibility.

C. Fluency.

In describing pseudopsychologies, it can be said that they... A. Have constantly changed over time as their followers have some new evidence. B. Have followers who are skeptical critics of their own theories. C. Give the appearance of science but are actually false. D. Are characterized by all of these statements.

C. Give the appearance of science but are actually false.

Recent experiments on eyewitnesses testimony have shown that the relationship between a person's confidence in his or her testimony and its accuracy... A.Depends upon whether the witness has at least a high school education. B. Is more suspect for children than for adults. C. Has almost no relationship. D. Depends upon whether the witness is male or female.

C. Has almost no relationship.

According to your textbook the future of psychotherapy will include... A. Less use of group therapies and self-help groups run by paraprofessionals. B. More therapy being provided by a psychiatrist and PhD- level practitioners. C. More precisely targeted medical therapies with fewer side effects. D. Greater use of long-term insight-oriented Therapies.

C. More precisely targeted medical therapies with fewer side effects.

Which of the following parts of the body would have the largest area represented on the motor area with regards to dexterity? A. Shoulder. B. Feet. C. Torso. D. Hand.

D. Hand.

Using IQ scores as the sole measure of giftedness... A. is the quickest way to identify creative individuals. B. ignores the poor adjustment that characterizes such persons. C. Ignores special talents. D. Identifies only persons from upper socioeconomic levels.

C. Ignores special talents.

When a person is lying, A. Illustrators increase, while emblems decrease. B. both illustrators and emblems decrease. C. Illustrators decrease, while emblems increase. D. both illustrators and emblems increase.

C. Illustrators decrease, while emblems increase.

A familiar disability occurs due to... A. A lack of oxygen at birth. B. Genetic abnormalities. C. Impoverished environments. D. Metabolic disorders.

C. Impoverished environments.

Jeanette was an extremely shy child when she was two years old. However, by the age five she was only mildly shy. This illustrates that... A. Inherited temperaments do not last after infancy. B. Shyness was not really a part of her temperament. C. Inherited temperament can be dynamically modified by learning. D. Inherited temperaments are lasting and cannot be modified by learning.

C. Inherited temperament can be dynamically modified by learning.

Abby's mother leaves the room. When her mother returns, Abby runs to greet her mother. However, when her mother reaches to pick her up, Abby pulls away and screams out "no." According to Mary Ainsworth's research, Abby is exhibiting a(n)... A. Separation anxiety disorder. B. Secure attachment. C. Insecure-ambivalent attachment. D. Insecure-avoidant attachment.

C. Insecure-ambivalent attachment.

After working to solve a problem for three hours, Larry has just about "given up" until he suddenly realizes the "perfect" solution and wonders why he did not think of it sooner. Larr has just experienced... A. Functional fixedness. B. Deductive thought. C. Insight. D. Inductive thought.

C. Insight.

Knowing a child's IQ at 11... A. Is both highly unreliable and highly invalid. B. Is a poor predictor of his or her IQ later in life. C. Is a good predictor of his or her IQ later in life. D. Tells little about the child present ability level.

C. Is a good predictor of his or her IQ later in life.

Reviewing and thinking about a problem before you go to sleep... A. Helps only imaginative people to solve problems. B. Is a guaranteed way to solve problems. C. Is a way to tap the creativity of dreams. D. Is of little value in problem-solving.

C. Is a way to tap the creativity of dreams.

Early maturation in boys... A. Creates less poised and more anxious individuals. B. Leads to less prestige among peers. C. Is generally beneficial. D. Is associated with the boys being less likely to get into trouble with drugs, alcohol, and antisocial behavior.

C. Is generally beneficial.

A neuron's threshold is... A. Another name for its negative after-potential. B. Another name for its synaptic potential. C. Its trigger point for firing. D. When a neuron is inactive.

C. Its trigger point for firing.

Jan and Pam are hoarders with each of their homes crammed with things acquired over two decades. Both ladies go into therapy. Jan's therapist helps Jan to better understand why she began collecting stuff, while Pam's therapist helps her to actively change her thoughts and beliefs about hoarding. The two ladies each went to which type of therapist? A. Jane went to a behavioral therapist, while Pam went to a humanistic therapist. B. Jan went to a cognitive therapist, while Pam went to a behavioral therapist. C. Jan went to a humanistic therapist, while Pam went to a cognitive therapist. D. Jan went to a cognitive therapist, while Pam went to a psychodynamic therapist,

C. Jan went to a humanistic therapist, while Pam went to a cognitive therapist.

Basic assumptions of behavior modification include which of the following? A. gaining insight Into one's problems brings relief from symptoms and ultimately changes one's Behavior. B. Emotional disturbances are the result of past experiences that must be understood and an resolve before personal growth can be achieved. C. Learned responses cause problems; therefore, Behavior can be changed by relearning. D. Emotional disturbances are caused by a loss of meaning in one's life.

C. Learned responses cause problems; therefore, Behavior can be changed by relearning.

Because psychology students tend to define "negative" as "bad," students often incorrectly define "negative reinforcement" as a punishment. This illustrates which barrier to problem-solving? A. Emotional. B. Perceptual. C. Learned. D. Cultural.

C. Learned.

In humans, the right side of the brain mainly controls the ______ of the body. A. Upper half. B. Right side. C. Left side. D. Lower half.

C. Left side.

Neuroscientists speculate that newborn humans (and monkeys) are able to imitate others because networks of specific types of neurons are activated when an infant watches someone perform an action with this same network being used to perform that action. These specific types of neurons are called _________ neurons. A. Effector. B. Reflector. C. Mirror. D. Sensory.

C. Mirror.

Lawrence Kohlberg is known for his research in the area of _______ development. A. Language. B. Physical. C. Moral. D. Motor

C. Moral.

Regarding child molesters, which of the following statements is FALSE? A. as children, child molesters themselves were often witnesses to or victims of sexual abuse. B. Many child molesters are rigid, passive, puritanical, or religious. C. Most child molesters are single older men. D. Child molester are often a relative, friend, or acquaintance of the victim's family.

C. Most child molesters are single older men.

The researcher stated that in his experiment, aggression will be described as "the number of times a frustrated individual insults the person who prevented work on the puzzle." This description is considered to be a(n)... A. Theoretical element. B. Estimated value. C. Operational definition. D. Hypothesis.

C. Operational definition.

Brain pathology related to brain diseases, brain injuries, lead or mercury poisoning, and damage due to drug use would be classified as ________ disorders. A. Mood. B. Somatoform. C. Organic mental. D. Dissociative.

C. Organic mental.

Frank considers himself to be a total failure in every aspect of his life because he failed one class. Brinks Behavior exemplifies the Distortion of thinking that cognitive therapist call... A. Selective perception. B. Incongruence. C. Overgeneralization. D. All-or-nothing thinking.

C. Overgeneralization.

When listening to your professor's lecture, interpreting the meaning of the professor's words is an example of... A. Assimilation. B. Accommodation. C. Perception. D. Sensations.

C. Perception.

Which of the following BEST describes the role of the typical father in child rearing? A. caregiver. B. Advocate. C. Playmate. D. Protector.

C. Playmate.

If a child whines and get his parents attention, he will tend to line more frequently in the future because he is experienced which principle of operant conditioning? A. Shaping. B. Nonreinforcement. C. Positive reinforcement. D. Punishment.

C. Positive reinforcement.

A person suffering from anxiety predicts that he will make a fool of himself at a social gathering; and, in fact he does. A psychologists predicts that students in his morning class will outperform those in his afternoon class and, in fact, they do. What do the anxious person and the psychologist have in common? They both... A. Are engaged in the Barnum effect. B. Possess a precognition type of ESP. C. Produced a self-fulfilling prophecy. D. Are engaged in the placebo effect.

C. Produced a self-fulfilling prophecy.

Theory building is important in psychology because it... A. Prevents excessive reliance on empiricism. B. Reduces the need for hypothesis testing. C. Provides explanations and guides future research. D. Relies heavily on naturalistic observation.

C. Provides explanations and guides future research.

A therapy in which clients act out personal conflicts and feelings in the presence of others who plays supporting roles is referred to as... A. Existentialism. B. Humanism. C. Psychodrama. D. Somatic therapy.

C. Psychodrama.

Erikson termed the conflicts one experiences between personal impulses and the social world... A. Moral dilemmas. B. Developmental milestones. C. Psychosocial dilemmas. D. Developmental tasks.

C. Psychosocial dilemmas.

A technique used to facilitate positive changes in a person's personality, Behavior, or personal adjustment defines... A. Pharmacotherapy. B. Somatotherapy. C. Psychotherapy. D. Psychosurgery.

C. Psychotherapy.

When the infant monkeys in the attachment study were frightened by rubber snakes, they... A. Ran to the surrogate mother that was the source of food. B. Cowered in the corner of the cage in a disturbed manner. C. Ran to the terry cloth surrogate mothers for security. D. Ran to the wire surrogate mother for security.

C. Ran to the terry cloth surrogate mothers for security.

A gene whose influence will be expressed only when it is paired with a second gene of the same type is known as a _________ gene. A. Gender-linked B. Dominant C. Recessive D. Polygenic

C. Recessive

Ronald is in the first grade and is given an addition worksheet to complete. At one point on the worksheet, the problems 2+5= and 5+2= are side-by-side. Ronald uses his fingers to count out each problem, although the same numbers are used but in a different order. Ronald is illustrating his lack of... A. Object permanence. B. Animism. C. Reversibility. D. Egocentrism.

C. Reversibility.

In Pavlov's experiment with dogs, the unconditioned response was the... A. Food. B. Bell. C. Salvation to the food. D. Salvation to the bell.

C. Salvation to the food.

Rhythms of seasons underlie the depression known as _______ disorder. A. Cyclothymic. B. Dysthymic. C. Seasonal affective. D. Bipolar.

C. Seasonal affective.

You have had a great deal of anxiety sufficient to cause you so unless I get discomfort and I've been urged by friends to consider seeking professional psychological help. Perhaps you should consider... A. Having yourself hospitalized to relieve your stress. B. Seeking advice from friends on how to eliminate your anxiety. C. Seeking professional psychological help. D. Doing nothing, as you are likely to feel better in a few days due to spontaneous remission.

C. Seeking professional psychological help.

When we rate words or concepts on dimensions, such as good/bad, strong/weak, and active/passive, we are measuring the connotative meaning of the words or concepts by using which approach? A. Cross-stimulation effect. B. Phonemic differential. C. Semantic differential. D. Random search strategy.

C. Semantic differential.

Which of the following statements about American Sign Language (ASL) is TRUE? A. ASL is understood by all those who use other forms of sign language. B. ASL is a symbolic communication that lacks syntax and grammar. C. Signing children pass through the same stages of language development at the same age as speaking children. D. American Sign Language is classified as a pantomimed code.

C. Signing children pass through the same stages of language development at the same age as speaking children.

Most people will say they were asleep within four minutes after which of the following appear? A. Beta waves. B. A hypnic jerk. C. Sleep spindles. D. Alpha waves.

C. Sleep spindles.

Which of the following skills would an amnesia patient be the most likely to learn? A. Learning a new telephone number. B. Remembering the names of three new people to which they were just introduced. C. Solving a complex puzzle in a normal amount of time. D. Relating current events that were read in the morning newspaper.

C. Solving a complex puzzle in a normal amount of time.

Hypochondriasis, pain disorders, and conversion disorders are types of _________ disorders. A. Psychosomatic. B. Dissociative. C. Somatoform. D. Organic mental.

C. Somatoform.

Dr. Orazzo is using an objective method of defining abnormality. However, this method will not tell her about the meanings of the deviations Chief tamed nor does it tell her where to draw the line between normality and abnormality. The method of defining that Dr. Orazzo is using is... A. social nonconformity. B. Cultural relativity. C. Statistical abnormality. D. mental nonconformity.

C. Statistical abnormality.

Recent life crisis, risk-taking, sudden mood swings, Rage/anger, seeking revenge, and no sense of purpose in life are among the warning signs for a(n)... A. social anxiety. B. Personality disorder. C. Suicide attempt. D. Affective psychosis.

C. Suicide attempt.

Regarding the performance of various ethnic groups and IQ test, which of the following statement is FALSE? A. the Assumption, biases, and the contents of standard IQ test do not always allow meaningful comparisons between ethnic, cultural, or racial groups. B. As a group, African Americans are more likely than European Americans to live in environment that are physically, educationally, and intellectually impoverished. C. The ethnic background of test creator has not been shown to affect the test performance of the test takers, whether the ethnicity was the same or was different from the test creator. D. One study found that place in poor African American children into European American adoptive families increase the children's IQs by an average of 13 points.

C. The ethnic background of test creator has not been shown to affect the test performance of the test takers, whether the ethnicity was the same or was different from the test creator.

Responding to a substance, such as a sugar pill or a saline injection, as if it were the real drug is referred to as... A. A psychosomatic complication. B. An anthropomorphic error. C. The placebo effect. D. The somatization variable.

C. The placebo effect.

Most male-female performance differences can be traced to... A. Genetic differences that cause men's overall IQ to be significantly higher than women's. B. Genetic differences they cause women's overall IQ to be significantly higher than men's C. The tendency for parents and Educators to encourage males and females to learn different skills. D. Hormonal differences that occur after puberty.

C. The tendency for parents and Educators to encourage males and females to learn different skills.

You decide to practice working the algebra problems that the teacher worked in class. You already know what the correct answer should be, so you work the problem with this answer in mind. This scenario illustrates... A. Convergence. B. Perceptual accommodation. C. Top-down processing. D. Bottom-up processing

C. Top-down processing.

The main technique for estimating how much heredity and environment affect intelligence involves the use of... A. Artificial intelligence. B. Culture-fair testing. C. Twin Studies. D. High-stakes testing.

C. Twin Studies.

You have just eaten a large meal and extremely full. Then, the waiter tells you about their signature dessert, "Death by Chocolate." Your ordering of this dessert is best explained by... A. an internal need. B. drive reduction. C. an incentive value. D. homeostasis.

C. an incentive value.

The process of suppressing and undesirable habit, such as smoking or drinking, so she a Ting it with a painful or uncomfortable stimulus is known as... A. resistance. B. Systematic desensitization. C. aversion therapy. D. Transference.

C. aversion therapy.

Females who have been exposed to androgens before birth tend to... A. show an elevated interest in male-oriented activities as adults. B. become permanently masculinized. C. be "tomboys" in childhood. D. have difficulty becoming pregnant.

C. be "tomboys" in childhood.

Operant conditioning focuses on how... A. people learn through the association of stimuli. B. People utilize their inborn reflexive habits. C. behavior is influenced by its consequences. D. Changes in behavior are related to mental events.

C. behavior is influenced by its consequences.

Cassie begins implementing a routine in which she keeps track of her calories, uses learning techniques to control her eating habits, and start exercising. We would say that Cassie has begun a... A. metabolism motivation program. B. modification of cognitive habitual processes. C. behavioral dieting program. D. self-esteem based dieting strategy.

C. behavioral dieting program.

Motives can be divided into three major categories, which are.... A. primary, secondary, and tertiary. B. physical, social, and individual, C. biological, stimulus, and learned. D. prepotent, homeostatic, and incentive.

C. biological, stimulus, and learned.

To estimate a child's Intelligence, we would need to compare the child's mental age to his or her... A. Average grade point average. B. nonverbal IQ. C. chronological age. D. present grade level.

C. chronological age.

An important way that psychologists work to decrease behavioral risks to health is through the use of... A. electroconvulsive shock therapy for unhealthy, anxious patients. B. sensory deprivation activities. C. community health campaigns that provide role models. D. jail time for people who violate health codes.

C. community health campaigns that provide role models.

In a laboratory experiments in which the dogs had developed learned helplessness, an effective method that helped the animals regain "hope" and feelings of control over the environment involved... A. having the dogs make "friends" with one of the experimenters. B. providing food treats every time they were shocked. C. forcibly "dragging" the dogs into the "safe" compartment. D. lowering the voltage of the shocks.

C. forcibly "dragging" the dogs into the "safe" compartment.

Concerning the candle problem present in your textbook, students who saw the box in which the materials were presented as only a container rather than as a part of the solutions were exhibiting... A. cognitive dissonance. B. mental consolidation. C. functional fixedness. D. Retroactive interference.

C. functional fixedness.

The process of learning gender behaviors regarded as appropriate for one's sec in a given culture is called... A. sexual scripting. B. sexual orientation. C. gender role socialization. D. biological biasing.

C. gender role socialization.

Lie detectors measure... A. a physically unique "lie response." B. muscular responses. C. general emotional arousal. D. feelings such as guilt and anger.

C. general emotional arousal.

Commitment proceedings are legal proceedings that may result in the finding of... A. psychosis. B. neurosis. C. insanity. D. psychopathology.

C. insanity.

Choosing among all the different makes, models, fuel-efficiencies, price ranges, and available options on the various automobiles would be an example of a(n) ___________ conflict. A. approach-avoidance. B. avoidance-avoidance. C. multiple approach-avoidance. D. approach-approach.

C. multiple approach-avoidance.

In a typical Catch-22 situation, a woman who are independent, aggressive, and unemotional may be considered unhealthy even though women who are vain, emotional, dependent and irrational (all feminine traits in our culture) may be classified as having a histrionic or dependent personality disorder. This example of gender bias is judging normality... A. occurs because women are genetically predisposed to emotional dysfunction more so than our men. B. occurs because the standards we set for males are higher than those for females. C. occurs because the standards for judging normality tend to be based on males. D. Occurred in the 1950s but does not occur today.

C. occurs because the standards for judging normality tend to be based on males.

A circadian rhythm refers to a cycle... A. of emotional arousal. B. during which various bodily systems are in dysfunctional state. C. of bodily activity approximately 24 hours in length. D. of sexual receptivity caused by hormone cycles.

C. of bodily activity approximately 24 hours in length.

A person who advocates eugenics would believe that... A. an interactionist view should be taken in which genetic and environmental factors are combined in the determination of traits. B. animal studies should be disregarded because their findings cannot be generalized to humans. C. one can selectively breed for desirable characteristics, such as intelligence. D. environment plays a larger role in the development of intelligence.

C. one can selectively breed for desirable characteristics, such as intelligence.

In order to discover whether the king had been cheated, Archimedes began by asking himself, "How can I tell what metals have been used in the crown without damaging it?" At this point, Archimedes is in which stage of creative thought? A. incubation. B. preparation. C. orientation. D. verification.

C. orientation.

Melinda fails a test because she chose to go to a party over the weekend rather than studying. However, Melinda tells everyone she failed the test because the teacher is "too hard" and "does not like me." Thus, the real source of Melinda's frustration is... A. due to pressure, although she thinks she is having an approach-avoidance conflict. B. nococial, while she thinks it is social. C. personal, while she perceives the frustration to be external. D. external, while she perceives the frustration to be personal.

C. personal, while she perceives the frustration to be external.

If 98 out of 100 people respond "Golden Retriever" when asked to name what dog best represents the concept "dog" than a golden retriever would be called a... A. feature. B. relational concept. C. prototype. D. denotative concept.

C. prototype.

You are hiking through a national forest when something moves under the leaves ahead of you and to your left. Can you then see the creature and determine that is indeed a poisonous snake. Deciding how you will react and avoid getting bit is known as... A. the adaptation phase. B. primary appraisal. C. secondary appraisal. D. the reactive plan stage.

C. secondary appraisal.

When Anthony gets a test paper back on which he earned an A, phrases like "Atta boy" and "Good going" pop into his head; and he feels a sense of pride in his accomplishments. This illustrates the social learning concept of... A. conditions of worth. B. unconditional positive regard. C. self-reinforcement. D. the ego ideal.

C. self-reinforcement.

To control your weight over the long-term, the brain monitors the amount of fat stored in your body in specialized fat cells, thus maintaining a(n)... A. weight standard. B. obesity limit. C. set point. D. critical weight.

C. set point.

The impact of long-term stresses can be understood by examining the body's defenses against stress, a pattern known as... A. the mind-body connection. B one's locus of control. C. the general adaptation syndrome. D. the defense mechanism model.

C. the general adaptation syndrome.

Concerning "honesty" strict learning theorists would say that... A. knowing a person is "honest" allows us to predict his/her behavior in a multitude of situations. B. "honesty" is considered a secondary trait. C. the specific situation would have to be known in order to make a "correct" prediction. D. "honesty" is the result of a lack of fixation during psychosocial development.

C. the specific situation would have to be known in order to make a "correct" prediction.

According to the contemporary view of emotion, which of the following gives rise to your initial autonomic nervous system arousal and the cognitive labeling of an emotional stimulus, such as a large, snarling dog? A. your facial feedback expression. B. your cognitive awareness of fear. C. your emotional appraisal of the situation. D. you adaptive behavior of running.

C. your emotional appraisal of the situation.

Average intelligence is usually defined as any score from... A. 120 to 129. B. 70 to 89. C. 110 to 119. D. 90 to 109.

D. 90 to 109.

At Mary's school, the teachers reward the desirable behaviors of their students by giving them "good cards" for each behavior. The students then exchanged the cards at the school " store" on Friday for candy, toys, or special privileges. This school is utilizing a therapeutic program known as... A. EDMR. B. systematic desensitization. C. Vicarious conditioning. D. A token economy.

D. A token economy.

Because Baby Eddie has a pet dog, he thinks all animals have four legs and a tail, bark and are furry. The first time he sees a cat, this theory works util the cat meows. Baby Eddie must now change his original idea. This change is Baby Eddie's thinking so that he now knows that some animals called dogs bark and some animals called cats meow illustrates... A. Object permanence. B. Assimilation. C. Abstraction. D. Accommodation.

D. Accommodation.

The grasping, rooting, and sucking reflexes of infants are best described as... A. Fixed action patterns. B. Instincts. C. Conditioned reflexes. D. Adaptive reflexes.

D. Adaptive reflexes.

What we remember depends on... A. What we pay attention to. B. How we elaborate our memory. C. What we feel strongly about and what we regard as important. D. All of these

D. All of these

Poverty has been shown to affect... A. A child's cognitive development. B. A child's socioemotional development. C. A child's health with poor children being sick more often. D. All of these.

D. All of these.

The reflex arc occurs when a stimulus provokes a(n)... A. Somatic response from the brain. B. Automatic response from the brain. C. Automatic response from a cranial nerve. D. Automatic response within the spinal cord without the brain involvement.

D. Automatic response within the spinal cord without the brain involvement.

Emily is a late maturing girl. Compared to the early maturing girls, we can expect Emily to... A. Have greater peer prestige and adult approval in junior high. B. Be more independent and more active in junior high. C. Be more likely to get in trouble in junior high. D. Be less at risk for premature identity formation.

D. Be less at risk for premature identity formation.

The central nervous system is composed of the... A. Sympathetic and parasympathetic systems. B. Autonomic and Somatic systems. C. Brain and somatic system. D. Brain and spinal cord.

D. Brain and spinal cord.

When presented with "The Porcupine and the Moles" story, the girls in Gilligan's story tended to use an ethic of... A. Justice. B. Common sense. C. Autonomy. D. Caring.

D. Caring.

Which of the following is NOT one of the six elements of the scientific method? A. Gathering evidence. B. Publishing results. C. Defining a problem. D. Consensus review.

D. Consensus review.

Skills that must be mastered or personal changes that must be made for optimal development, such as learning to read, adjusting to sexual maturity, or establishing a vocation are best described as... A. Developmental milestones. B. Moral dilemmas. C. Psychosocial dilemmas. D. Developmental tasks.

D. Developmental tasks.

You have just witnessed what appears to be the funnel cloud of a tornado off in the distance, your body is physically aroused with these bodily stirrings causing you to be "moved" to take action. You are experiencing a(n) A. Motive. B. Drive. C. Expectation. D. Emotion.

D. Emotion.

Hormones are poured into the bloodstream and lymph system by the glands of the _____ system. A. Reticular activating. B. Limbic. C. Somatic. D. Endocrine.

D. Endocrine.

Nature is to heredity as nurture is to... A. Maturation B. Readiness. C. Genetics. D. Environment.

D. Environment.

Which hormone tends to arouse the body and is associated with fear? A. Melatonin. B. Estrogen. C. Norepinephrine. D. Epinephrine.

D. Epinephrine.

When one can alternately see both a wine glass and then two faces within the same image, this illustrates the reversibility of_____ images. A. Continuation. B. Proximity. C. Closure. D. Figure-ground.

D. Figure-ground.

When you made a poor grade on history in high school, you made sure you read the chapters twice before taking the next test. Although this strategy has NOT worked in college, you continue this INEFFECTIVE strategy, disregarding the instructors study advice. Your continued poor performance on test is due to... A. Functional incubation. B. Consolidation. C. Disjunctive thinking. D. Fixation.

D. Fixation.

Which of the following psychologists proposed that the simplest organization involves grouping some sensations into an object or figure, that stands out on a plainer background... A. Humanistic psychologists. B. Psychodynamic psychologists. C. Structuralists. D. Gestalt psychologists.

D. Gestalt psychologists.

Initiative is fostered by... A. Acquiring a stable and consistent identity. B. Learning a sense of industry. C. A child's experience with toilet training. D. Giving a child the freedom to play, to use imagination, and to choose activities.

D. Giving a child the freedom to play, to use imagination, and to choose activities.

A verbal interaction between a trained mental health professional and several clients at the same time is called... A. Meta-analysis. B. Individual therapy. C. Insight therapy. D. Group therapy.

D. Group therapy.

If Marty is exhibiting maladaptive Behavior, we would expect him to... A. exhibit behavior that is culturally relative but statistically rare. B. Show social nonconformity behavior that may or may not be statistically rare. C. Show a lack of consistent happiness and sociability in his life. D. Having underlying psychological or biological dysfunction that makes it difficult for him to meet the demands of day-to-day life.

D. Having underlying psychological or biological dysfunction that makes it difficult for him to meet the demands of day-to-day life.

Kristen is a 24-year-old college graduate still living at home, never married, has no children, and no settled career. Kristen most likely is... A. Caring for her elderly parents. B. Experiencing premature identity formation. C. Exhibiting generativity. D. In a social status known as emerging adulthood.

D. In a social status known as emerging adulthood.

You offer to trade your nickel for a dime held by a child. The child accepts the trade because the nickel is bigger. The child is... A. In the neonatal stage of the growth sequence. B. Displaying reversibility. C. Obviously past the age of reason. D. In the preoperational stage.

D. In the preoperational stage.

A commonsense approach to psychology... A. Is the most reliable. B. Provides information that is specific and consistent. C. Is the basis for most psychological theories. D. Is often contradicted by empirical evidence.

D. Is often contradicted by empirical evidence.

Jan and her husband Dean are going on a vacation to the beach. Jan wants to lie on the beach and read a good book, while Dean wants to scuba dive and surf. Which of the following statements is TRUE? A. Jan is fixated at an earlier stage of development, while Dean is self-actualized. B. Jan is a high sensation seeker, while Dean is a low sensation seeker. C. Jan is self-actualized, while Dean is fixated at an earlier stage of development. D. Jan is a low sensation seeker, while Dean is a high sensation seeker.

D. Jan is a low sensation seeker, while Dean is a high sensation seeker.

Jasmine enjoys cooking and dreams of opening her own restaurant, while Jacob cooks at a local restaurant and dreams of the day he can save enough money to quit his job and go back to college. Regarding cooking, A. both are exhibiting extrinsic motivation. B. both are exhibiting intrinsic motivation. C. Jasmine is exhibiting extrinsic motivation, while Jacob is exhibiting intrinsic motivation. D. Jasmine is exhibiting intrinsic motivation, while Jacob is exhibiting extrinsic motivation.

D. Jasmine is exhibiting intrinsic motivation, while Jacob is exhibiting extrinsic motivation.

Which of the following would be POOR advice for a person who wants to lead a self-actualized life? A. Take responsibility for every aspect of your life. B. Examine your motives. C. Get involved and commit to a "calling" in life. D. Judge your progress by the standards of others.

D. Judge your progress by the standards of others.

An effective way for athletes to improve their skills off the field is to practice by rehearsing... A. the use of synaesthesia regarding their upcoming athletic performance. B. negative self-talk to motivate themselves. C. relaxation images to use when they make a "bad" play. D. Kinesthetic images of themselves performing flawlessly.

D. Kinesthetic images of themselves performing flawlessly.

Major disturbances in emotion, such as depression or mania, are classified as _________ disorders. A. Anxiety. B. Dissociative. C. Schizophrenic. D. Mood.

D. Mood.

A general conclusion that one might draw from the relationship between Zulu experiences and their perception of the Muller-Lyer illusion is that... A. Some principles of perception are universal. B. Perception has a more powerful effect on experience than experience on perception. C. People cannot trust their own senses because they can never provide completely reliable knowledge. D. Past experiences and perceptual habits play a role in determining human perception.

D. Past experiences and perceptual habits play a role in determining human perception.

Studies of people who have had their vision restored after a lifetime of blindness found that it takes time for these people to learn to organize their sensations into... A. Sensory sets. B. Sensory Habituations. C. Realizations. D. Perceptions.

D. Perceptions.

Regarding the use of behavior therapy, which of the following statements is FALSE? A. The use of most behavior therapy is quite complicated and requires a great deal of expertise. B. Although a person should seek professional help when a significant problem exists, lesser difficulties may be treated by applying behavior principles to oneself. C. In covert sensitization, imagery is used to reduce the occurrence of an undesirable response. D. Rapid smoking is in a version technique that smokers can carry out on their own.

D. Rapid smoking is in a version technique that smokers can carry out on their own.

After spending spending time in stage 4, the sleeper... A. awakens directly from stage 4. B. Returns directly to stage 1. C. Returns directly to stage 2. D. Returns to Stage 3, then Stage 2, then Stage 1.

D. Returns to Stage 3, then Stage 2, then Stage 1.

Four-year-old Annie is trying to put together her first jigsaw puzzle and has a great deal of difficulty until her father gives her some hints, such as "try putting the corners together first" and "that area looks like it needs pink ones, so lets find all the pink pieces." Annie's father is using the process Vygotsky called... A. Parentese. B. Vicarious learning. C. Forced learning. D. Scaffolding.

D. Scaffolding.

Regarding mood disorders, which of the following statements is TRUE? A. Overall all men are 50% more likely to become depressed than women. B. Postpartum depression is relatively mild and last just a few days. C. If one identical twin is depressed, the other has a 97% chance of suffering from depression with the probability of the fraternal twins being 50%. D. Scientist believe that the brain chemicals of Serotonin, noradrenaline, and dopamine are involved in mood disorders.

D. Scientist believe that the brain chemicals of Serotonin, noradrenaline, and dopamine are involved in mood disorders.

When sensations in the visual, kinesthetic, and vestibular systems do not match, which of the following has occurred? A. Synesthesia. B. Sensory integration. C. Sensory selection. D. Sensory conflict.

D. Sensory conflict.

The psychiatrist who viewed emotional problems, which he called "Hysteria" as originating from unconscious conflicts was... A. Viktor Frankl. B. Fritz Perls. C. Carl Rogers. D. Sigmund Freud.

D. Sigmund Freud.

Juan has claustrophobia so Juan's therapist gradually exposes Juan to smaller and smaller places while Juan relaxes to each step in the hierarchy, beginning with the lowest fear-producing item. Juan's phobia is being treated with... A. Aversive conditioning. B. confrontation. C. rational-emotive behavior therapy. D. Systematic desensitization.

D. Systematic desensitization.

When making diagnoses and conducting therapy, psychologist should... A. Use the behavior of the middle-class American male as a standard for assessing what is "average." B. Usa an unstated standard for judging what is normal for each individual. C. Ignore cultural differences and use one psychological perspective for viewing all clients. D. Take into account their clients' personal differences regarding age, race, culture, ethnicity, gender, and sexual orientation.

D. Take into account their clients' personal differences regarding age, race, culture, ethnicity, gender, and sexual orientation.

Negative attention-seeking will develop in some children because... A. Punishment is always an effective and teaches aggression. B. The parents reward the children's correct social behaviors in only limited situations. C. Punishment was not used to get rid of the negative attention-seeking and the children. D. The children will receive attention when they are ill-behaved but are ignored when they are well-behaved.

D. The children will receive attention when they are ill-behaved but are ignored when they are well-behaved.

As consumption of ice cream increases during the year, so let's have the number of abrogated assault. A politician Advocates that ice-cream be banned so that the crime rate will go down. Besides needing to be voted out of office, this politician needs to understand... A. The difference between observation and interference. B. The dangers of a single example. C. the Barnum effect occurs only in 10 year Cycles. D. The difference between the correlation and causation.

D. The difference between the correlation and causation.

A doctor activates the surface of a patient's brain by touching it with a small electrified wire and the patient tells the doctor what effect the stimulation had. Regarding this procedure, which of the following statements is TRUE? A. This procedure is impossible because patients cannot be awake during brain surgery. B. The doctor is using an EEG to "turn on" brain structures. C. The doctor is performing an ablation. D. The doctor is using an electrode to "turn on" brain structures.

D. The doctor is using an electrode to "turn on" brain structures.

A cognitive therapist would view unhealthy anxiety as... A. a loss of meaning in one's life. B. A raging conflict among the id, ego, and superego. C. The effects of learning and conditioning. D. The result of distorted thinking.

D. The result of distorted thinking.

Regarding masturbation, which of the following statements is TRUE? A. People are only compelled to masturbate because they lack a sex partner. B. Masturbation occurs because of immaturity and low self-esteem. C. Masturbation is presently seen as an abnormal sexual behavior. D. They only negative effects of masturbation come from learned attitudes.

D. They only negative effects of masturbation come from learned attitudes.

During the formal operations stage, children begin to... A. Think primarily about concrete objects or situations. B. Accurately use concepts of time, space, and number. C. Develop the concept of object permanency. D. Think in terms of abstract principles and hypothetical possibilities.

D. Think in terms of abstract principles and hypothetical possibilities.

The term "cognition" includes... A. Eidetic imagery and processing. B. Classical and instrumental conditioning. C. An active process by which sensory input is selected, organized, and integrated. D. Thinking, problem solving, reasoning, and daydreaming.

D. Thinking, problem solving, reasoning, and daydreaming.

One day, a classmate with whom you've been trying to go out socially gives you his/her number on a napkin (it was the only thing you had handy at the time). Unfortunately, when you wash your jeans, you forget to take out the napkin, and now all you can piece together of the phone number are the first six digits. If you go to the phone and dial the first six digits plus "0", then dial the first six plus "1", then dial the first six digits plus "2", etc. until you dial the correct number, you are solving the problem by... A. Insight. B. Using heuristics. C. Rote. D. Trial and error.

D. Trial and error.

On which section of the Stanford-Binet (SB5) are subjects asked to correctly remember the order of colored beads seen, to repeat a series of digits forward and backward, and to name the last word in each sentence after hearing several sentences read? A. Visual-Spatial processing. B. Fluid reasoning. C. Quantitative reasoning. D. Working memory.

D. Working memory.

Genuine self-esteem is based on a(n) __________ appraisal of your strengths and weaknesses. A. balanced. B. high positive. C. modest. D. accurate.

D. accurate.

Johannes Gutenberg, creator of the printing press, realized while at a wine harvest that the mechanical pressure used to crush grapes could also be used to imprint letters on paper, which illustrates that incubation is especially fruitful when you... A. develop a prototype though convergent thinking. B. are allowed time to "thin-slice" possible solutions. C. can create mental sets from external cues. D. are exposed to external cues that relate to the problem.

D. are exposed to external cues that relate to the problem.

It is important to realize that many people who do transmit STDs are initially... A. androgynous. B. androgential. C. non-infectious. D. asymptomatic.

D. asymptomatic.

Attitudes toward sexual behavior in the U.S. have... A. become less tolerant of sexual behavior of all kinds. B. Changed little in the last 50 years. C. resulted in major increases in extramarital sexual contacts. D. become more tolerant, especially of other people's behavior.

D. become more tolerant, especially of other people's behavior.

More American college students than Chinese students were able to solve the cave problem by recalling the "Hansel and Gretel story," while more Chinese students than American students solved the statue's weight problem by recalling the "weighing of the elephant" fable. These differing percentages illustrate all of the following EXCEPT that... A. every culture prepares its members to solve some types of problems more easily than others. B. the culture we grow up in affects our ability to use selective comparision to solve problems. C. being exposed to a similar problem in the past helps the students' problem solving. D. both groups of students exhibited functional fixedness regarding the stories they had heard in childhood.

D. both groups of students exhibited functional fixedness regarding the stories they has heard in childhood.

Upsetting thoughts and emotions may have a negative influence on health through their link with the... A. thalamic stress projection system. B. stage of alarm and resistance. C. sensory and psychomotor systems. D. brain and immune system.

D. brain and immune system.

Margo, a young nurse, realizes with dismay that she has "lost all patience with her patients" and wishes they would "go somewhere else to be sick." Margo's feelings are a clear sign of... A. conflict. B. pressure. C. frustration. D. burnout.

D. burnout.

Sydney has wanted to be teacher all of her life. For the first 5 years of teaching, she spent every summer, holiday, and weekend trying to come up with new activities for her students. However, she now feels that all of her efforts were in vain and that the students do not want to learn and do not appreciate anything she tries to do to help them learn. Sydney feels "used up," apathetic, and detached from her students. Sydney is experiencing... A. conflict. B. pressure. C. anxiety. D. burnout.

D. burnout.

Which of the following skills would be most difficult for an artificial intelligence system to do? A. predict the weather. B. beat a world champion chess player. C. harmonize music. D. decipher a sentence of misspelled words.

D. decipher a sentence of misspelled words.

Going from general principles to specific situations is called ________ thought. A. inductive. B. illogical. C. creative. D. deductive.

D. deductive.

According to Steven Johnson, today's video games, the internet, and TV... A. have increased stress level, while having no effect on our intelligence. B. demand less cognitive effort and thus decrease our intelligence. C. have increased stress levels, while decreasing our intelligence. D. demand greater cognitive effort and thus increase our intelligence.

D. demand greater cognitive effort and thus increase our intelligence.

According to Freud, the _________ is executive of the personality and operates on the reality principle, delaying action until it is practical or appropriate. A. super ego. B. id. C. psyche. D. ego.

D. ego.

At the highest skill levels, experts performers tend to do all of the following EXCEPT... A. exhibit automatic processing. B. rise above rules and plans. C. exhibit exceptional memories. D. exhibit quick, fluid, and insightful decisions, thinking, and actions.

D. exhibit quick, fluid, and insightful decisions, thinking, and actions.

Mild college-related depression can be helped by doing all of the following EXCEPT... A. making a daily schedule for yourself and filling up every hour of the day with something to do. B. challenging self-critical thinking. C. learning to manage one's college work. D. getting more than nine hours of sleep as well as taking an afternoon nap to increase your energy.

D. getting more than nine hours of sleep as well as taking an afternoon nap to increase your energy.

The branch of psychology that studies how behavioral principles can be used to prevent illness and promote health is known as _________ psychology. A. developmental. B. stress. C. behavioral. D. health.

D. health.

Which of the following theories stress private, subjective, experience, and personal growth? A. behavioral theories. B. social learning theories. C. psychodynamic theories. D. humanistic theories.

D. humanistic theories.

An ideal situation for studying the role of genetics on behavior is... A. fraternal twins reared together. B. fraternal twins reared apart. C. identical twins reared together. D. identical twins reared apart.

D. identical twins reared apart.

Which of the following best describes the maladaptive behaviors included in the term "psychopathology"? A. loss of contact with reality. B. Loss of contact with reality and compulsive behaviors. C. loss of contact with reality compulsive behaviors, and drug addiction. D. loss of contact with reality compulsive behaviors, drug addiction, and behaviors that limit personal growth.

D. loss of contact with reality compulsive behaviors, drug addiction, and behaviors that limit personal growth.

Many of history's renowned artists, writers, poets, and composers, including Vincent Van Gogh, Edgar Allan Poe, Emily Dickinson, and Ernest Hemmingway most likely suffered from... A. paranoid schizophrenia. B. dissociative disorders. C. obsessive-compulsive disorder. D. mood disorders.

D. mood disorders.

If we describe Dan as being sociable, orderly, and intelligent, we are referring to his... A. personality style. B. temperamental character. C. personality type. D. personality traits.

D. personality traits.

The functions of hospice services include... A. keeping the dying person quiet, protected, and free from decision-making. B. Relieving the family of the difficulty of caring for a dying person. C. Providing the latest technology in life-sustaining measures. D. providing the dying with support, guidance, pain relief, and companionship.

D. providing the dying with support, guidance, pain relief, and companionship.

Which of the following disorders is characterized by the person being socially withdrawn, being unable to control his or her thoughts or actions, and by the presence of delusions and/or hallucinations? A. neurotic disorders. B. dissociative disorders. C. somatoform disorders. D. psychotic disorders.

D. psychotic disorders.

The main difference between self-actualization and the other needs in Maslow's hierarchy of motives is... A. that more people are motivated by self-actualization than by all the other needs. B. that self-actualization must occur before self-esteem needs can be met. C. that basic needs are less dominant than self-actualization. D. that self-actualization is not a deficiency motive.

D. that self-actualization is not a deficiency motive.

The best thing for someone to do if they encounter an exhibitionist is to... A. yell at the person and chastise him for his deviant behavior. B. shout or scream loudly to scare the person away. C. become visibly upset so the person will feel remorseful about his behavior. D. try not to act shocked or alarmed because this further encourages the behavior.

D. try not to act shocked or alarmed because this further encourages the behavior.

Those who doubt that animals use language would base their doubts on animals' inability to... A. communicate within their own species. B. use gestures to communicate with others species. C. use American Sign Language. D. use language in productive ways.

D. use language in productive ways.

When you are feeling excessively hassled from the ordinary stresses of life, it is a good idea to... A. pretend like you are really not stressed. B. throw yourself into work and keep very busy. C. drink moderate amounts of alcohol or get a prescription for tranquilizers. D. use stress management techniques.

D. use stress management techniques.

A major disadvantage of projective tests is that... A. they are little use to clinical psychologists. B. scoring is highly involved and overly objective. C. they are unstructured. D. validity and objectivity are typically low.

D. validity and objectivity are typically low.

Displaced aggression is said to occur... A. whenever stress is channeled into socially acceptable activities. B. when a person watches an aggressive sporting event. C. whenever stress is channeled into aggressive activities. D. when aggression is directed at a target other than the source of frustration.

D. when aggression is directed at a target other than the source of frustration.

Mental imagery can involve all of the following EXCEPT... A. the mental rotation of the imaged object. B. picture-like visual images. C. images for smell, taste, and pain. D. words and categories of objects.

D. words and categories of objects.

Four-year-old Aaron has been asleep for a little over an hour when its mother hears him scream out. When she gets to his bedroom, little Aaron is sitting up in bed in a state of panic. When his mother attempts to console him, she notes that his pajamas are drenched in sweat, and he is only semi awake. After this episode, Aaron goes back to sleep. His mother surprise the next morning that Aaron only vaguely remembers his mother coming into the room. Aaron most likely experienced... A. A night terror. B. Narcolepsy. C. Sleep apnea. D. A nightmare.

A. A night terror.

Psychologists define learning as... A. A relatively permanent change in behavior due to experience. B. A temporary change in behavior due to one's motivational state. C. Behavioral changes as a result of maturation and development. D. The modification and replacement of old behaviors and habits with socially acceptable behaviors.

A. A relatively permanent change in behavior due to experience.

You come home from work and find that your dog has dug up the tree you just planted. To reduce this digging, you would be best advised to punish the dog... A. As the dog begins to dig. B. As soon as you get home and discover the hole. C. Several hours after digging. D. By taking the dog to each hole he has dug recently and then punishing him.

A. As the dog begins to dig.

While taking an exam, you read a question and know that you know the answer to that question but you just cannot retrieve it during the exam. However, the answer pops in your head as you leave the classroom following the exam. This is an example of when a memory is... A. Available but not accessible. B. Temporary rather than permanent. C. Short-term but not long-term. D. Explicit but not implicit.

A. Available but not accessible.

The purported ability to perceive events or gain information in ways that appear unaffected by distance or normal physical barriers is called... A. Clairvoyance. B. Precognition. C. Telepathy. D. Psychokinesis.

A. Clairvoyance.

Sound energy is converted to nerve impulses by the hair cells inside the... A. Cochlea. B. Tympanic membrane. C. Otolith organs. D. Pinna.

A. Cochlea.

Mina is ten years old and is undergoing chemotherapy. When she goes to into the treatment center for her fourth treatment, Mina experiences nausea before the treatment even begins. Her nausea had been triggered by the sight of the treatment room. In this example, the treatment room was the... A. Conditioned stimulus. B. Unconditioned stimulus C. Conditioned response D. Unconditioned response

A. Conditioned stimulus.

You enter the dark theater from the brightly lighted lobby; and you are temporarily unable to see where to walk. After a short time, you can see the entire room as your retinas have undergone.... A. Dark adaptation. B. Cone enhancement. C. The Ishihara effect. D. Retinal disparity.

A. Dark adaptation.

As you read this chapter in Psychology, you frequently asked yourself "why" questions, such as "Why would that be true?" as well as relating this new information with your own experiences involving your own "memory." You are using which of the following to help you remember the information in this chapter? A. Elaborative processing. B. Chunking or recoding. C. Maintenance rehearsal. D. Redintegration.

A. Elaborative processing.

While studying the concept of reinforcement, Erica asked herself how reinforcement and punishment differ and what kinds of reinforcement (and punishment) there are; and she tried to think of personal examples of the use of reinforcement. Erica is using... A. Elaborative processing. B. Rote learning. C. Maintenance rehearsal. D. Chunking or recoding.

A. Elaborative processing.

Which type of memory is about the "what," "where," and "when" of our lives and make it possible for us to mentally travel back in time and re-experience events? A. Episodic memory. B. Eidetic memory. C. Semantic memory. D. Echoic memory.

A. Episodic memory.

The order of sensitivity to the four basic taste sensations may have... A. Helped prevent poisonings. B. Helped prevent malnutrition. C. Been connected with one's performance. D. Led to cooked food.

A. Helped prevent poisonings.

Which of the following parts of the brain works as the "switching station" between short term and long term memories? A. Hippocampus. B. Frontal lobes. C. Cerebellum. D. Broca's area

A. Hippocampus.

To modify their own nightmares, nightmare sufferers can use... A. Imagery rehearsal. B. Dream sublimation. C. Secondary displacement. D. Dream condensation.

A. Imagery rehearsal.

Cue-dependent theories of memory suggest that you would do best on you history test if you could be test... A. In the room where you studied. B. In a quiet room with only a small group of students. C. With a large group of history majors. D. With students who are your age and share your interests.

A. In the room where you studied.

John's memory of events before his brain operation are clear; and he can carry on normal conversations with others. However, he cannot form new lasting memories, so John lives "externally" in the present. Therefore, which of the following statements would be FALSE regarding John's condition? A. John's surgery significantly damaged the space and time capacity of a short-term memory. B. John cannot consolidate new information. C. John has anterograde amnesia. D. John most likely has damage to his hippocampus.

A. John's surgery significantly damaged the space and time capacity of a short-term memory.

Jose studies, stays awake for eight hours, then sleeps for eight hours. Santos studies, then sleeps eight hours, and is then awake for eight hours. Regarding the amount of forgetting that each of these students will experience... A. Jose will forget more of what he studied than Santos. B. Santos will forget more of what he studied than Jose. C. Both will forget the Same amount of material. D. Neither will forget a significant amount of material since only 16 hours have passed since they studied.

A. Jose will forget more of what he studied than Santos.

According to your text, if you want to eliminate a nightmare, you should... A. Mentally rehearse the dream you want. B. Change your eating habits. C. Increase your sleeping time. D. Decrease your sleeping time.

A. Mentally rehearse the dream you want.

As a visitor to England, you as an American, have difficulty remembering to drive and the left side of the road as they do and to look in the opposite way before crossing the street. This illustrates... A. Negative transfer. B. Negative reinforcement. C. Modeling. D. Avoidance conditioning.

A. Negative transfer.

Advertisers often try to use higher order conditioning by... A. Pairing images which evoke good feelings with pictures of their products. B. Sounding loud tones at key points in the advertisement. C. Reducing fear or anxiety as they repeatedly show the same commercial. D. Repeating the same slogan over and over again.

A. Pairing images which evoke good feelings with pictures of their products.

Fatima sees her four-year-old son spontaneously picking up his toys. If she wishes for her son to continue this behavior, she should... A. Praise her son immediately after he finishes picking up the toys. B. Wait and praise her soon after his father gets home from work. C. Not praise her son because he will then expect praise every time he acts "good." D. Wait and only praise him if he picks them up again on another day.

A. Praise her son immediately after he finishes picking up the toys.

Smelling apple pie causes you to have vivid and detailed memories of the happy times that you spend at your grandmother's kitchen. This process is called... A. Redintegration. B. Eidetic imagery. C. Chunking or recoding. D. Cognitive learning.

A. Redintegration.

You are asked to learn the information presented in a psychology chapter, and you study for four hours. Then three months later, you are asked to study the same chapter information, and you finish your study in only two hours. In this case, your memory of having learned this information three months ago is shown through... A. Relearning. B. Recognition. C. Recall. D. Redintegration.

A. Relearning.

The best way to differentiate operant conditioning from classical conditioning is to decide if the... A. Response is voluntary or involuntary. B. Stimulus is conditioned or unconditioned. C. Consequence comes before or after the response. D. Response is pleasant or unpleasant.

A. Response is voluntary or involuntary.

Converting important features of the world into neural messages understood by the brain is called... A. Sensory coding. B. Sensory gating. C. Sensory accommodation. D. Sensory adaptation.

A. Sensory coding.

At the National Cheerleading contest, all the contestants are seated in one area within the stadium. Of the 1,000 students are seated in this area, your eyes discern about 100 grapes do to each group of cheerleaders having different color outfits. The way you grouped the students is most likely due to the perceptual principle of... A. Similarity. B. Nearness. C. Contihuity. D. Continuity.

A. Similarity.

In the textbook example regarding "Shaping a Teacher," psychology students would look interested in the teacher's lecture when she was standing on the right side of the room; and they would look bored when she was standing on the left side. The students were shaping the teacher's behavior using _________ Reinforcers... A. Social. B. Primary. C. Token. D. Cognitive.

A. Social

Research shows that people who learned a list of words while in a happy mood we called them better when they were again happy. People who are in the list while sad remembered best when they were said. This illustrates... A. State-dependent learning. B. Eidetic imagery. C. Suppression. D. Regression imagery.

A. State-dependent learning.

Janice broke up with her long-time boyfriend Chuck two months ago. Every time any memory of Chuck creeps into her mind, she consciously pushes these memories out of her mind. Janice is trying to forget "this guy" by using... A. Suppression. B. Repression. C. Disuse. D. Interference.

A. Suppression.

In an experiment, the discovery learning group of students was encouraged to see that "piece" of parallelogram could be "moved" to create a rectangle, while the rote learning group memorized the height times base formula the results of the experiment demonstrated that... A. The discovery learning group performed better and the unusual problems. B. The rote learning group perform better on the unusual problems C. Both groups performed exceptionally well and unusual problems D. Both groups performed poorly on unusual problems

A. The discovery learning group performed better and the unusual problems.

When you are about to get a shot with a hypodermic needle, your muscles tighten and there is a catch in your breathing because you have learned to expect that getting poked with a needle will be painful. This explanation of your behavior is consistent with... A. The informational view. B. The interactionism theory. C. The law of effect. D. Shaping.

A. The informational view.

Which of the following are tangible secondary reinforcers, such as money, gold stars, and poker chips that can be exchanged for food, lodgings, and other necessities... A. Token reinforcers. B. Social reinforcers. C. Unconditioned reinforcers. D. Negative reinforcers.

A. Token reinforcers.

A college professor who gives daily quizzes, grades them immediately, and returns them with the errors and corrected answers shown is... A. Using feedback. B. Using the CAI technique. C. Applying instructional games. D. Employing the two-factor theory of learning.

A. Using feedback.

Acquiring a fear of Thunder by watching your parents react to it in a fearful way illustrates... A. Vicarious classical conditioning. B. Instrumental conditioning. C. Response chaining. D. Vicarious withdrawal.

A. Vicarious classical conditioning.

Your eyes transduce only a tiny fraction of the entire range of electromagnetic energies, which we call the... A. Visible spectrum. B. Biological transduction range. C. Spectral analysis limitation. D. Perceptual limit.

A. Visible spectrum.

Regarding the experiments using intracranial self-stimulation of the pleasure centers, which of the following statements is false... A. When given the choice, rats preferred food, water or sex to pressing the bar to receive brain stimulation. B. Some of the rats constantly pressed the lever to get brain stimulation for 15 to 20 mins hours, collapsing from exhaustion. C. Psychoactive drugs, such as alcohol and cocaine, activate the same pleasure pathways that are activated by intracranial self-stimulation. D. Humans have been "wired" for brain stimulation experimentally to restrain uncontrollable outbursts of violence.

A. When given the choice, rats preferred food, water or sex to pressing the bar to receive brain stimulation.

When short-term memory (STM) is combined with other mental processes, it acts more like a "mental scratchpad" where we do much of our thinking. This aspect of STM is known as _______ memory. A. Working. B. Eidetic. C. Procedural. D. Integrative.

A. Working.

In a recent experiment, participants watch the movie in which a man and a woman are both on screen telling different stories simultaneously. The researchers asked them to focus on just one of the speakers while they monitored what was going on in their brains. Researchers found some areas, like those involving and hearing, continue to respond to both voices. But other parts of the brain, like those devoted to language, appear to respond only to the selected speaker. Afterward, volunteers who focused on the man had no trouble remembering that he did not like to clean his room. But they didn't recall anything about the women's parrot. This experiment demonstrates... A. Sensory conflict theory. B. Cocktail party effect. C. Habituation. D. Somesthetic sensations

B. Cocktail party effect.

Which of the following processes is like writing your name in wet concrete, that is, once the concrete is set, the information (your name) is fairly lasting; but while it is setting, it can be wiped out (amnesia) or scribbled over (interference)? A. Memory localization. B. Consolidation. C. Mnemonic processing. D. Priming.

B. Consolidation.

Which of the following best describes the unusual events that occurred in Pavlov's laboratory that led him to the discovery of classical conditioning? A. Dogs salivated after meat powder was placed in their mouths. B. Dogs sometimes salivated before the meat powder was placed in their mouths. C. Salvation existed in dogs as an unlearned reflex. D. Dogs salivated if and only if they were given a reward.

B. Dogs sometimes salivated before the meat powder was placed in their mouths.

A basic problem with the trichromatic theory is that... A. Color cannot be analyzed into "either-or" messages. B. Four colors seem to be primary (cannot get them by mixing other colors). C. Only two types of light-sensitive visual pigments have been found in the retina. D. It only explains events recorded in the optic pathways after information leaves the eyes.

B. Four colors seem to be primary (cannot get them by mixing other colors).

Sensory memory is the stage of memory that... A. Holds small amounts of information when one is thinking or problem-solving. B. Holds an exact record of incoming information for a few seconds or less. C. Involves the permanent storage of meaningful information. D. Involves the storage of conditioned responses and learned skills.

B. Holds an exact record of incoming information for a few seconds or less.

The fading or weakening of a memory trace due to a passage of time is called... A. Redintegration. B. Memory decay. C. Interference. D. Repression.

B. Memory decay.

According to their serial position effect, you are most likely to forget which items on a list... A. First. B. Middle. C. Last. D. First and last.

B. Middle.

Using punishment can be "habit forming" because putting a stop to someone else's irritating behavior can... A. Positively reinforce the person who applied the punishment. B. Negatively reinforce the person who applied the punishment. C. Encourage the person to use punishment less often in the future. D. Lead to the liberal use of positive reinforcers.

B. Negatively reinforce the person who applied the punishment.

The part of the ear that acts as a funnel to concentrate sounds and guide them into the ear canal is called the... A. Cochlea. B. Pinna C. Tympanic membrane. D. Auditory ossicle.

B. Pinna

In order to prevent SIDS, healthy babies should be... A. Placed face down to sleep. B. Placed on their backs to sleep. C. Propped up on a small pillow to sleep. D. Covered with a down comforter to prevent developing a chill while sleeping.

B. Placed on their backs to sleep.

In a classical experiment, Little Albert, a very young boy, was conditioned to be afraid of a white rat. He also became fearful of white furry rabbits and bearded men. This is an example of... A. Stimulus discrimination. B. Stimulus generalization. C. Extinction. D. Shaping.

B. Stimulus generalization.

In classical conditioning, learning is evident when a... A.Stimulus automatically elicits a consequence. B. Stimulus, which did not initially produce a response, now elicits that response. C. Spontaneously emitted response increases in frequency because of its consequences. D. Subject repeats an action he or she has observed another person perform.

B. Stimulus, which did not initially produce a response, now elicits that response.

Adolescents are especially susceptible to addiction because... A. Children and adolescents release more neurotransmitters than adults. B. The brain systems that restrain risk-taking are not as mature as those that reward pleasure-seeking. C. Of the hormones that are released at puberty that interact with the drugs to produce a more intensified and pleasurable effect. D. The Corpus callosum in the brains of adolescents are not fully developed.

B. The brain systems that restrain risk-taking are not as mature as those that reward pleasure-seeking.

If you watch television every night and want to study more, make it a rule not to turn on the set until you have studied for an hour. Then, lengthen the requirement each week. Using the television watching as a reinforcement for studying is an example of... A. Latent learning. B. The premack principle. C. Higher order conditioning. D. The law of Association.

B. The premack principle.

In their experiments on the brain and perceptual features, Blakemore, Cooper, Grobstein, and Chow found that... A. The kittens raised in the horizontally-striped rooms had difficulty jumping onto a chair in the normal environment. B. There was an actual decrease in brain cells tuned to the missing features. C. The kittens raised in the vertically-striped rooms ran into chair legs in the normal environment. D. All of these findings were true.

B. There was an actual decrease in brain cells tuned to the missing features.

A perceptual expectancy is... A. The organization of perceptions be beginning with low-level features. B. The information surrounding a stimulus that gives meaning to the stimulus. C. A readiness to perceive in a particular manner. D. An ingrained pattern of perceptual organization and attention.

C. A readiness to perceive in a particular manner.

Perceptions can be misconstructed as they are filtered through... A. Our expectations. B. Our attitudes, values, and beliefs. C. All of these. D. Our needs.

C. All of these.

Because our sense select, analyze, and filter, information until the only most important information remains our senses are considered ______ systems. A. Convergent. B. Somesthetic. C. Data reduction. D. Replicating.

C. Data reduction.

If you like to study while watching television or instant messaging, beware because dividing your attention like that often leads to forgetting due to... A. Disuse. B. Memory decay. C. Encoding failure. D. Redintegration.

C. Encoding failure.

Because figure-ground organization is the first perceptual ability to appear after cataract patients regain their sight, it... A. Is a type of perceptual habituation. B. Involves retinal disparity. C. Is probably inborn. D. Involves bottom-up processing.

C. Is probably inborn

According to the partial reinforcement effect, acquiring a response through partial reinforcement makes the response... A. Weaker. B. Easier to learn. C. More resistant to extinction. D. Less resistant to extinction.

C. More resistant to extinction.

Creative people Habituate... A more often during creative production. B. More rapidly than average. C. More slowly than average. D. To unfamiliar stimuli.

C. More slowly than average.

According to the dual process theory of sleep, which of the following "sharpens" our memories of the previous day's more important experiences? A. Delta waves. B. NREM sleep. C. REM sleep. D. Somnambulism.

C. REM sleep.

In a classical experiment, two groups of rats explored a maze with the first group being rewarded with food at the end of the maze, while the second group wandered the maze but were not rewarded with food at the end of the maze. On another trail, the second group was rewarded with food at the end of the maze. This second group of "uneducated" rats... A. Only then began to learn the maze. B. Showed only a slightly slower completion time. C. Ran the maze as quickly as the first group. D. Never learned the maze until numerous feeding trails were utilized.

C. Ran the maze as quickly as the first group.

Essay questions, fill-in-the-blanks, and the recitation of poems all measure memory through... A. Relearning. B. Recognition. C. Recall. D. Retroactive redintegration.

C. Recall.

You are training a rat to press a bar. Instead of waiting for the Rats first accidental bar press, you reinforce the rat with a food pellet for facing the bar and then turning toward the bar. Next, you reinforce the rat every time he takes a step toward the bar, until he touches the bar. Lastly, you reinforce the wrap for pressing the bar. You are... A. Using latent learning to train the rat. B. Feeding the rats behavior. C. Shaping the rats behavior. D. Using secondary reinforcement with the rat.

C. Shaping the rats behavior.

Through behavioral therapy, Jennifer was able to extinguish her fear of dogs and to actually pet a dog without tensing up. However, Jennifer has a new neighbor with the large dog they growled at her the first time to try to pet it. Now, Jennifer's fearfulness towards all dogs has returned. This reoccurrence of her past fear is an example of... A. Latent learning. B. Response chaining. C. Spontaneous recovery. D. Stimulus control.

C. Spontaneous recovery.

ou are walking on a gravel road. As you look down at your feet, you can make out the various shapes, sizes, colors, and jagged edges of the rocks, but as you look at the road in the distance, the rocks appear smaller and smoother. This differing view of the rocks illustrates... A. Interposition. B. Accommodation. C. Texture Gradients. D. Linear perspective.

C. Texture Gradients.

A dog whistle can be heard by dogs but not by humans. This whistle takes advantage of a characteristic of sensory systems known as... A. The difference threshold. B. A just noticeable difference. C. The absolute threshold. D. Perceptual defense.

C. The absolute threshold.

According to the trichromatic theory of color visions... A. The eye has color receptors that are limited to white, black, and red. B. The brain separates images into "either-or" messages of red-green, blue-yellow, or black-white. C. There are three types of cones, each of sensitive to either red, green or blue. D. There are three types of rods, each of sensitive to either red, green or blue.

C. There are three types of cones, each of sensitive to either red, green or blue.

A teen who is punished at home a lot will be gone as much as possible. This is an example of... A. Classical conditioning. B. Modeling. C. Negative transfer. D. Avoidance learning.

D. Avoidance learning.

Sleep is characterized by all of the following EXCEPT for... A. Being an innate biological rhythm. B. Giving way temporarily in times of great danger. C. Being necessary for like in humans. D. Being a total loss of consciousness.

D. Being a total loss of consciousness.

Prolonged sleep loss can cause a major disruption of mental and emotional functioning characterized by hallucinations, delusions, and confusion called temporary... A. Narcolepsy. B. Sleep apnea. C. Sleep deprivation psychosis. D. Chronic insomnia.

D. Chronic insomnia.

Memory, thinking, problem-solving, and the use of concepts and language are all involved in... A. Operant learning. B. Instrumental Conditioning. C. Response-dependent learning. D. Cognitive learning.

D. Cognitive learning.

Sense receptors are biological transducers, which means that they... A. Select from a broad range of stimulation. B. Detect only a limited range of energies. C. Analyze the environment into important features. D. Convert one form of energy to another.

D. Convert one form of energy to another.

The reason that sleeptalking makes little sense and why sleepwalkers are confused and remember little when awakened is that both sleep disturbances occur during... A. REM sleep. B. Cataplexic episodes. C. Hypnopompic episodes. D. Deep NREM sleep.

D. Deep NREM sleep.

Jessie had a nagging backache after a day spent raking leaves. This nagging pain... A. Involves the body's reminding system for pain. B. May cause agony long after this injury has healed. C. Is carried by small nerve fibers. D. Involves all of these characteristics.

D. Involves all of these characteristics.

A skeptic of psi abilities is one who... A. Shows uncritical acceptance of psi events. B. Is against the continuation of parapsychology research. C. Rarely experiences the "decline effect." D. Is unconvinced regarding the existence of psi events.

D. Is unconvinced regarding the existence of psi events.

In one experiment, children watched either live models or films of a person or cartoon character hitting an inflatable doll ("Bo-Bo the Clown" doll). What happen when they had the opportunity to play with the same doll? A. Few of the children imitated the behaviors they had seen. B. Those who saw the live model imitated the behavior, but those who saw the cartoon did not. C. Those who saw the cartoon model imitated the behavior, but those who saw the live models did not. D. Most of the children imitated the behaviors they had seen.

D. Most of the children imitated the behaviors they had seen.

Any event that increases the probability that a response will occur again is referred to as a... A. Motive. B. Consequence. C. Feedback stimulus. D. Reinforcement.

D. Reinforcement.

You are taking a psychology test and are trying to remember what you studied. The memory process being used when you try to remember the psychology material is... A. Encoding. B. Feedback. C. Storage rehearsal. D. Retrieval.

D. Retrieval.

If you cram for a psychology exam and then later the same night cram for a History exam, your memory for psychology will be less accurate because of... A. Disuse. B. Memory Decay. C. Proactive interference. D. Retroactive interference.

D. Retroactive interference.

Which of the following determines what information movers from sensory memory to short-term memory? A. Consolidation B. Constructive processing C. Priming D. Selective attention

D. Selective attention

It has been found that what you eat can affect how easily you go to sleep. For example, eating which of the following types of foods increases the amount of tryptophan that reaches the brain and causes sleepiness? A. Leafy green vegetables. B. Citrus fruit juices. C. Proteins. D. Starchy foods.

D. Starchy foods.

Each time he steps up to the plate, a baseball player touches the bill of his cap three times, just as he did before hitting a homerun in a previous game. Which principle accounts for his actions? A. Modeling. B. Reminiscence. C. Positive transfer. D. Superstitious behavior.

D. Superstitious behavior.

You and your husband go to the movies. About 20 minutes into the movie, he asks you who the actress is that just came on the screen and if she is on some television show you watch. You know that you know the name but you cannot recall it this illustrates... A. The savings score. B. Suppression State. C. Encoding failure. D. The tip-of-the-tongue state.

D. The tip-of-the-tongue state.

A teacher tells her first graders that whenever the bell clock rings, the students who are in their seats will each receive one token to put in their "happy jars." She then says, "No one knows when the clock will ring. It might be ten minutes, it might be 17 minutes; so you better be in your seat." The teacher is using which schedule of reinforcement for "in-seat" behavior... A. Fixed ratio. B. Fixed Interval. C. Variable ratio. D. Variable interval

D. Variable interval

Regarding long-term memories, which of the following statements is FALSE? A. Gaps in one's memory are common. B. It is possible to believe that you are remember things that never happened. C. As new long-term memories are stored, older memories are often updated, changed, lost, or revised. D. When we fill in gaps in memory with logic, guessing, or new information, it is referred to as chunking or recoding.

D. When we fill in gaps in memory with logic, guessing, or new information, it is referred to as chunking or recoding.


Conjuntos de estudio relacionados

Physical Science Study Guide Chapter 12.3/12.4 Work And Energy

View Set

1340 PU ch 26: Caring for Clients with Cardiac Dysrhythmias

View Set

ASSESS Chapter 6 Chapter Problems

View Set

CORE MVC:::: C#/.NET/MVC/Core, ASP .Net Core Web Applications MVC, ASP.NET Core MVC 2, Spring Core, MVC, AOP, ASP Net Core MVC RESTful, C# MVC 5 / Core 2.0, ASP.NET Core MVC, ASP.NET Core MVC, ASP.NET Core MVC, ASP.NET Core MVC

View Set

Chapter 12: Sexual Orientation, Identity and Expressions:

View Set

THE NERVOUS SYSTEM- ATI TEAS NOTES

View Set

Chapter 11: Maternal Adaptation During Pregnancy (Prep U)

View Set

random practice APUSH questions (AP test review)

View Set